You are on page 1of 214

1.

2 Definitions and Fundamental Ideas of Thermodynamics


As with all sciences, thermodynamics is concerned with the mathematical modeling of the real world. In order that the mathematical deductions are consistent, we need some precise definitions of the basic concepts. The following is a discussion of some of the concepts we will need. Several of these will be further amplified in the lectures and in other handouts. If you need additional information or examples concerning these topics, they are described clearly and in-depth in (SB&VW). They are also covered, although in a less detailed manner, in Chapters 1 and 2 of the book by Van Ness.

1.2.1 The Continuum Model


Matter may be described at a molecular (or microscopic) level using the techniques of statistical mechanics and kinetic theory. For engineering purposes, however, we want ``averaged'' information, i.e., a macroscopic, not a microscopic, description. There are two reasons for this. First, a microscopic description of an engineering device may produce too much information to manage. For example, contains of air at standard temperature and pressure molecules (VW, S & B:2.2), each of which has a position and a

velocity. Typical engineering applications involve more than molecules. Second, and more importantly, microscopic positions and velocities are generally not useful for determining how macroscopic systems will act or react unless, for instance, their total effect is integrated. We therefore neglect the fact that real substances are composed of discrete molecules and model matter from the start as a smoothed-out continuum. The information we have about a continuum represents the microscopic information averaged over a volume. Classical thermodynamics is concerned only with continua.

1.2.2 The Concept of a ``System''


A thermodynamic system is a quantity of matter of fixed identity, around which we can draw a boundary (see Figure 1.3 for an example). The boundaries may be fixed or moveable. Work or heat can be transferred across the system boundary. Everything outside the boundary is the surroundings. When working with devices such as engines it is often useful to define the system to be an identifiable volume with flow in and out. This is termed a control volume. An example is shown in Figure 1.5.

A closed system is a special class of system with boundaries that matter cannot cross. Hence the principle of the conservation of mass is automatically satisfied whenever we employ a closed system analysis. This type of system is sometimes termed a control mass.

Figure 1.3: Piston (boundary) and gas (system)

Figure 1.4: Boundary around electric motor (system)

Figure 1.5: Sample control volume

1.2.3 The Concept of a ``State''


The thermodynamic state of a system is defined by specifying values of a set of measurable properties sufficient to determine all other properties. For fluid systems, typical properties are pressure, volume and temperature. More complex systems may require the specification of more unusual properties. As an example, the state of an electric battery requires the specification of the amount of electric charge it contains. Properties may be extensive or intensive. Extensive properties are additive. Thus, if the system is divided into a number of sub-systems, the value of the property for the whole system is equal to the sum of the values for the parts.

Volume is an extensive property. Intensive properties do not depend on the quantity of matter present. Temperature and pressure are intensive properties. Specific properties are extensive properties per unit mass and are denoted by lower case letters. For example:

Specific properties are intensive because they do not depend on the mass of the system. The properties of a simple system are uniform throughout. In general, however, the properties of a system can vary from point to point. We can usually analyze a general system by sub-dividing it (either conceptually or in practice) into a number of simple systems in each of which the properties are assumed to be uniform. It is important to note that properties describe states only when the system is in equilibrium.

Muddy Points Specific properties (MP 1.1) What is the difference between extensive and intensive properties? (MP 1.2)

1.2.4 The Concept of ``Equilibrium''


The state of a system in which properties have definite, unchanged values as long as external conditions are unchanged is called an equilibrium state.

[Mechanical Equilibrium]

[Thermal Equilibrium]

Figure 1.6: Equilibrium A system in thermodynamic equilibrium satisfies: 1. mechanical equilibrium (no unbalanced forces) 2. thermal equilibrium (no temperature differences) 3. chemical equilibrium.

1.2.5 The Concept of a ``Process''


If the state of a system changes, then it is undergoing a process. The succession of states through which the system passes defines the path of the process. If, at the end of the process, the properties have returned to their original values, the system has undergone a cyclic process or a cycle. Note that even if a system has returned to its original state and completed a cycle, the state of the surroundings may have changed.

1.2.6 Quasi-Equilibrium Processes


We are often interested in charting thermodynamic processes between states on thermodynamic coordinates. Recall from the end of Section 1.2.3, however, that properties define a state only when a system is in equilibrium. If a process involves finite, unbalanced forces, the system can pass through non-equilibrium states, which we cannot treat. An extremely useful idealization, however, is that only ``infinitesimal'' unbalanced forces exist, so that the process can be viewed as taking place in a series of ``quasi-equilibrium'' states. (The term quasi can be taken to mean ``as if;'' you will see it used in a number of contexts such as quasione-dimensional, quasi-steady, etc.) For this to be true the process must be slow in relation to the time needed for the system to come to equilibrium internally. For a gas at conditions of interest to us, a given molecule can undergo roughly molecular collisions per second, so that, if ten collisions are needed to come to equilibrium, the equilibration time is on the order of seconds. This is generally much shorter than the time scales associated with the bulk properties of the flow (say the time needed for a fluid particle to move some significant fraction of the length of the device of interest). Over a large range of parameters, therefore, it is a very good approximation to view the thermodynamic processes as consisting of such a succession of equilibrium states, which we can chart. [VW, S& B: 2.3-2.4]

The figures below demonstrate the use of thermodynamics coordinates to plot isolines, lines along which a property is constant. They include constant temperature lines, or isotherms, on a diagram, constant volume lines, or -

isochors on a - diagram, and constant pressure lines, or isobars, on a diagram for an ideal gas.

Real substances may have phase changes (water to water vapor, or water to ice, for example), which we can also plot on thermodynamic coordinates. We will see such phase changes plotted and used for liquid-vapor power generation cycles in Chapter 8. A preview is given in Figure 1.15 at the end of this chapter.

Figure:

diagram

Figure: - diagram Figure: - diagram Figure 1.7: Thermodynamics coordinates and isolines for an ideal gas

1.2.7 Equations of state


It is an experimental fact that two properties are needed to define the state of any pure substance in equilibrium or undergoing a steady or quasi-steady process. [VW, S & B: 3.1, 3.3]. Thus for a simple compressible gas like air,

where is the volume per unit mass, , etc.

. In words, if we know

and

we know

Any of these is equivalent to an equation , which is known as an equation of state. The equation of state for an ideal gas, which is a very good approximation to real gases at conditions that are typically of interest for aerospace applications1.2, is

where

is the volume per mol of gas and .

is the ``Universal Gas Constant,''

A form of this equation which is more useful in fluid flow problems is obtained if we divide by the molecular weight, :

where R is

, which has a different value for different gases due to the .

different molecular weights. For air at room conditions,

1.3 Changing the State of a System with Heat and Work


Changes in the state of a system are produced by interactions with the environment through heat and work, which are two different modes of energy transfer. During these interactions, equilibrium (a static or quasi-static process) is necessary for the equations that relate system properties to one-another to be valid.

1.3.1 Heat
Heat is energy transferred due to temperature differences only. 1. Heat transfer can alter system states; 2. Bodies don't ``contain'' heat; heat is identified as it comes across system boundaries; 3. The amount of heat needed to go from one state to another is path dependent; 4. Adiabatic processes are ones in which no heat is transferred.

1.3.2 Zeroth Law of Thermodynamics


With the material we have discussed so far, we are now in a position to describe the Zeroth Law. Like the other laws of thermodynamics we will see, the Zeroth Law is based on observation. We start with two such observations:

1. If two bodies are in contact through a thermally-conducting boundary for a sufficiently long time, no further observable changes take place; thermal equilibrium is said to prevail. 2. Two systems which are individually in thermal equilibrium with a third are in thermal equilibrium with each other; all three systems have the same value of the property called temperature. These closely connected ideas of temperature and thermal equilibrium are expressed formally in the ``Zeroth Law of Thermodynamics:'' Zeroth Law: There exists for every thermodynamic system in equilibrium a property called temperature. Equality of temperature is a necessary and sufficient condition for thermal equilibrium. The Zeroth Law thus defines a property (temperature) and describes its behavior1.3. Note that this law is true regardless of how we measure the property temperature. (Other relationships we work with will typically require an absolute scale, so in these notes we use either the Kelvin or Rankine

scales. Temperature scales will be discussed further in Section 6.2.) The zeroth law is depicted schematically in Figure 1.8.

Figure 1.8: The zeroth law schematically

1.3.3 Work
[VW, S & B: 4.1-4.6]

Section 1.3.1 stated that heat is a way of changing the energy of a system by virtue of a temperature difference only. Any other means for changing the energy of a system is called work. We can have push-pull work (e.g. in a piston-cylinder, lifting a weight), electric and magnetic work (e.g. an electric motor), chemical work, surface tension work, elastic work, etc. In defining work, we focus on the effects that the system (e.g. an engine) has on its surroundings. Thus we define work as being positive when the system does work on the surroundings (energy leaves the system). If work is done on the system (energy added to the system), the work is negative. Consider a simple compressible substance, for example, a gas (the system), exerting a force on the surroundings via a piston, which moves through some distance, (Figure 1.9). The work done on the surroundings, , is

therefore

Why is the pressure

instead of

? Consider

(vacuum). No work is

done on the surroundings even though changes.

changes and the system volume

Use of instead of is often inconvenient because it is usually the state of the system that we are interested in. The external pressure can only be related to the system pressure if . For this to occur, there cannot be any friction, and the process must also be slow enough so that pressure differences due to

accelerations are not significant. In other words, we require a ``quasi-static'' process, . Consider .

Therefore, when static),

is small (the process is quasi-

and the work done by the system is the same as the work done on the surroundings. Under these conditions, we say that the process is ``reversible.'' The conditions for reversibility are that: 1. If the process is reversed, the system and the surroundings will be returned to the original states. 2. To reverse the process we need to apply only an infinitesimal .A reversible process can be altered in direction by infinitesimal changes in the external conditions (see Van Ness, Chapter 2). Remember this result, that we can only relate work done on surroundings to system pressure for quasi-static (or reversible) processes. In the case of a ``free expansion,'' where (vacuum), is not related to to the work) because the system is not in equilibrium. (and thus, not related

We can write the above expression for work done by the system in terms of the specific volume, ,

where is the mass of the system. Note that if the system volume expands against a force, work is done by the system. If the system volume contracts under a force, work is done on the system.

Figure 1.9: A closed system (dashed box) against a piston, which moves into the surroundings

Figure 1.10: Work during an irreversible process For simple compressible substances in reversible processes, the work done can be represented as the area under a curve in a pressure-volume diagram, as in Figure 1.11(a).

[Work is area under curve of

[Work depends on

path] Figure 1.11: Work in Key points to note are the following:

coordinates

1. Properties only depend on states, but work is path dependent (depends on the path taken between states); therefore work is not a property, and not a state variable. 2. When we say the path. , the work between states 1 and 2, we need to specify

3. For irreversible (non-reversible) processes, we cannot use the work must be given or it must be found by another method.

; either

Muddy Points How do we know when work is done? (MP 1.3)

1.3.3.1 Example: Work on Two Simple Paths


Consider Figure 1.12, which shows a system undergoing quasi-static processes for which we can calculate work interactions as .

Figure 1.12: Simple processes

Along Path a:

Along Path b:

Practice Questions Given a piston filled with air, ice, a bunsen burner, and a stack of small weights, describe 1. how you would use these to move along either path a or path b above, and 2. how you would physically know the work is different along each path.

1.3.3.2 Example: Work Done During Expansion of a Gas


Consider the quasi-static, isothermal expansion of a thermally ideal gas from to , , as shown in Figure 1.13. To find the work we must know the path. Is it specified? Yes, the path is specified as isothermal. ,

Figure 1.13: Quasi-static, isothermal expansion of an ideal gas The equation of state for a thermally ideal gas is

where is the number of moles, is the Universal gas constant, and is the total system volume. We write the work as above, substituting the ideal gas equation of state,

also for

, so the work done by the system is

or in terms of the specific volume and the system mass,

1.3.4 Work vs. Heat - which is which?


We can have one, the other, or both: it depends on what crosses the system boundary (and thus, on how we define our system). For example consider a resistor that is heating a volume of water (Figure 1.14):

Figure 1.14: A resistor heating water 1. If the water is the system, then the state of the system will be changed by heat transferred from the resistor. 2. If the system is the water and the resistor combined, then the state of the system will be changed by electrical work.

1.4 Muddiest Points on Chapter 1


MP 1..1 Specific properties Energy, volume, enthalpy are all extensive properties. Their value depends not only on the temperature and pressure but also on ``how much,'' i.e., what the mass of the system is. The internal energy of two kilograms of air is twice as much as the internal energy of one kilogram of air. It is very often useful to work in terms of properties that do not depend on the mass of the system, and for this purpose we use the specific volume, specific energy, specific enthalpy, etc., which are the values of volume, energy, and enthalpy for a unit mass (kilogram) of the substance. For a system of mass , the relations between the two quantities are: , , .

MP 1..2 What is the difference between extensive and intensive properties? Intensive properties are properties that do not depend on the quantity of matter. For example, pressure and temperature are intensive properties. Energy, volume and enthalpy are all extensive properties. Their value depends on the mass of the system. For example, the enthalpy of a certain mass of a gas is doubled if the mass is doubled; the enthalpy of a system that consists of several parts is equal to the sum of the enthalpies of the parts. MP 1..3 How do we know when work is done? A rigorous test for whether work is done or not is whether a weight could have been raised in the process under consideration. I will hand out some additional material to supplement the notes on this point, which seems simple, but can be quite subtle to unravel in some situations.

[]

[]

[]

Figure 1.15: Pressure-temperature-volume surface for a substance that expands on freezing (from VW, S & B: 3.7)

2.1 First Law of Thermodynamics


[VW, S & B: 2.6] Observation leads to the following two assertions: 1. There exists for every system a property called energy, . The system energy can be considered as a sum of internal energy, kinetic energy, potential energy, and chemical energy. 1. Like the Zeroth Law, which defined a useful property, ``temperature,'' the First Law defines a useful property called ``energy.'' 2. The two new terms (compared to what you have seen in physics and dynamics, for example) are the internal energy and the chemical energy. For most situations in this class, we will neglect the chemical energy. We will generally not, however, neglect the internal energy, . It arises from the random or disorganized motion of molecules in the system, as shown in Figure 2.1. Since this molecular motion is primarily a function of temperature, the internal energy is sometimes called ``thermal energy.''

Figure 2.1: Random motion is the physical basis for internal energy 3. The internal energy, , is a function of the state of the system. Thus , or , or . Recall that for pure substances the entire state of the system is specified if any two properties are specified. (We will discuss the equations that relate the internal energy to these other variables as the course progresses.) 2. The change in energy of a system is equal to the difference between the heat added to the system and the work done by the system , (2..1) 3.

4. where

is the energy of the system,

is the heat input to the system, and is the work done by the system. (thermal energy) + 1. Like the Zeroth Law, the First Law describes the behavior of the new property [VW, S& B: Chapter 5]. 2. The equation can also be written on a per unit mass basis 3. 4. In many situations the potential energy, kinetic energy, and chemical energy of the system are constant or not important. Then 5. 6. and 7. 8. Note that and are not functions of state, but , which arises from molecular motion (see above), depends only on the state of the system; does not depend on how the system got to that state. We therefore have the striking result that: 9. 10. Sometimes this difference is emphasized by writing the First Law in differential form, (2..2) 11. 12. where the symbol `` '' is used to denote that these are not exact differentials but rather are dependent on path. 13. Note that the signs are important:

is defined to be positive if it is transferred to the system; thus the numerical value we substitute for will be positive

if heat is transferred to the system from the surroundings, and negative if heat is transferred from the system to the surroundings. [VW, S & B: 4.7-4.8] is defined to be positive if it is done by the system (see Section 1.3); thus the numerical value we substitute for will be positive if the system is doing work, and negative if work is being done on the system. [VW, S& B: 4.1-4.4] ,

14. For quasi-static processes we can substitute 15.

To give an example of where the first law is applied, consider the device shown in Figure 2.2. We heat a gas, it expands against a weight, some force (pressure times area) is applied over a distance, and work is done. The change in energy of the system supplies the connection between the heat added and work done. We will spend most of the course dealing with various applications of the first law -- in one form or another.

Figure 2.2: The change in energy of a system relates the heat added to the work done The form of the first law we have given here is sometimes called the ``control mass'' form, because it is well suited to dealing with systems of a fixed mass. We will see in Section 2.5 that this form can be written for a control volume with mass flow in and mass flow out (like a jet engine for example). We will call this the ``control volume'' form of the first law [VW, S & B: 5.8-5.12].

Muddy Points

What are the conventions for work and heat in the first law? (MP 2.1) When does ? (MP 2.2)

2.2 Corollaries of the First Law


1. Work done in any adiabatic ( ) process is a function of state. We can write the first law, setting the heat transfer term equal to zero, as (2..3) 2. 3. Since depends only on the state change, now function of the state change. can be found as a

Figure 2.3: The change in energy between two states is not path dependent. 4. For a cyclic process heat and work transfers are numerically equal

Figure 2.4: Since energy is a function of state only, any process that returns a system to its original state leaves its energy unchanged. 6. 7. therefore 8. 9. and

2.3 Example Applications of the First Law to motivate the use of a property called ``enthalpy''
[VW, S & B: 5.4-5.5]

2.3.1 Adiabatic, steady, throttling of a gas (flow through a valve or other restriction)
Figure 2.5 shows the configuration of interest. We wish to know the relation between properties upstream of the valve, denoted by ``1'' and those downstream, denoted by ``2''.

Figure 2.5: Adiabatic flow through a valve, a generic throttling process

Figure 2.6: Equivalence of actual system and piston model To analyze this situation, we can define the system (choosing the appropriate system is often a critical element in effective problem solving) as a unit mass of gas in the following two states. Initially the gas is upstream of the valve and just through the valve. In the final state the gas is downstream of the valve plus just before the valve. The figures on the left of Figure 2.6 show the actual configuration just described. In terms of the system behavior, however, we could replace the fluid external to the system by pistons which exert the same pressure that the external fluid exerts, as indicated schematically on the right side of Figure 2.6. The process is adiabatic, with changes in potential energy and kinetic energy assumed to be negligible. The first law for the system is therefore

The work done by the system is

Use of the first law leads to

In words, the initial and final states of the system have the same value of the quantity . For the case examined, since we are dealing with a unit .

mass, the initial and final states of the system have the same value of We define this quantity as the ``enthalpy,'' usually denoted by ,

In terms of the specific quantities, the enthalpy per unit mass is It is a function of the state of the system. of Joules per kilogram. has units of Joules, and has units

The utility and physical significance of enthalpy will become clearer as we work with more flow problems. For now, you may wish to think of it as follows (Levenspiel, 1996). When you evaluate the energy of an object of volume , you have to remember that the object had to push the surroundings out of the way to make room for itself. With pressure on the object, the work required to make a

place for itself is . This is so with any object or system, and this work may not be negligible. (The force of one atmosphere pressure on one square meter is equivalent to the force of a mass of about .) Thus the total energy of a body is its internal energy plus the extra energy it is credited with by having a volume at pressure . We call this total energy the enthalpy, .

Muddy Points

When is enthalpy the same in initial and final states? (MP 2.3)

2.3.2 Quasi-Static Expansion of a Gas


Consider a quasi-static process of constant pressure expansion. We can write the first law in terms of the states before and after the expansion as

and writing the work in terms of system properties,

By grouping terms we can write the heat input in terms of the enthalpy change of the system:

2.3.3 Transient filling of a tank


Another example of a flow process, this time for an unsteady flow, is the transient process of filling a tank, initially evacuated, from a surrounding atmosphere, which is at a pressure Figure 2.7. and a temperature . The configuration is shown in

Figure 2.7: A transient problem - filling of a tank from the atmosphere

At a given time, the valve at the tank inlet is opened and the outside air rushes in. The inflow stops when the pressure inside is equal to the pressure outside. The tank is insulated, so there is no heat transfer to the atmosphere. What is the final temperature of the gas in the tank? This time we take the system to be all the gas that enters the tank. The initial state has the system completely outside the tank, and the final state has the system completely inside the tank. The kinetic energy initially and in the final state is negligible, as is the change in potential energy, so the first law again takes the form

Work is done on the system, of magnitude the system, so

, where

is the initial volume of

In terms of quantities per unit mass ( mass of the system), The final value of the internal energy is

, where

is the

For a perfect gas with constant specific heats (see the next section, Section 2.4),

The final temperature is thus roughly

hotter than the outside air!

It may be helpful to recap what we used to solve this problem. There were basically four steps: 1. 2. 3. 4. Definition of the system Use of the first law Equating the work to a `` '' term Assuming the fluid to be a perfect gas with constant specific heats.

A message that can be taken from both of these examples (as well as from a large number of other more complex situations, is that the quantity occurs naturally in problems of fluid flow. Because the combination appears so frequently, it is not only defined but also tabulated as a function of temperature and pressure for a number of working fluids.

Muddy Points In the filling of a tank, why (physically) is the final temperature in the tank higher than the initial temperature? (MP 2.4)

2.3.4 The First Law in Terms of Enthalpy


We start with the first law in differential form and substitute assuming a quasi-static or reversible process: for by

The definition of enthalpy,

can be differentiated (applying the chain rule to the

term) to produce

Substituting the

above for the

in the First Law, we obtain

or

2.4 Specific Heats: the relation between temperature change and heat
[VW, S& B: 5.6] How much does a given amount of heat transfer change the temperature of a substance? It depends on the substance. In general (2..4)

where is a constant that depends on the substance. We can determine the constant for any substance if we know how much heat is transferred. Since heat is path dependent, however, we must specify the process, i.e., the path, to find . Two useful processes are constant pressure and constant volume, so we will consider these each in turn. We will call the specific heat at constant pressure , and that at constant volume , or and per unit mass.

1. The Specific Heat at Constant Volume Remember that if we specify any two properties of the system, then the state of the system is fully specified. In other words we can write , or . [VW, S & B: 5.7]

Consider the form changes with respect to

, and use the chain rule to write how and : (2..5)

For a constant volume process, the second term is zero since there is no change in volume, . Now if we write the First Law for a quasi-static process, with , (2..6)

we see that again the second term is zero if the process is also constant volume. Equating (2.5) and (2.6) with canceled in each,

and rearranging

In this case, any energy increase is due only to energy transfer as heat. We can therefore use our definition of specific heat from Equation (2.4) to define the specific heat for a constant volume process,

2. The Specific Heat at Constant Pressure

If we write , and consider a constant pressure process, we can perform a similar derivation to the one above and show that

In the derivation of , we considered only a constant volume process, hence the name, ``specific heat at constant volume.'' It is more useful, however, to think of in terms of its definition as a certain partial derivative, which is a thermodynamic property, rather than as a quantity related to heat transfer in a special process. In fact, the derivatives above are defined at any point in any quasi-static process whether that process is constant volume, constant pressure, or neither. The names ``specific heat at constant volume'' and ``specific heat at constant pressure'' are therefore unfortunate misnomers; and are thermodynamic properties of a substance, and by definition depend only the state. They are extremely important values, and have been experimentally determined as a function of the thermodynamic state for an enormous number of simple compressible substances2.1. To recap:

or

Practice Questions Throw an object from the top tier of the lecture hall to the front of the room. Estimate how much the temperature of the room has changed as a result. Start by listing what information you need to solve this problem.

2.4.1 Specific Heats of an Ideal Gas


The equation of state for an ideal gas is

where is the number of moles of gas in the volume . Ideal gas behavior furnishes an extremely good approximation to the behavior of real gases for a wide variety of aerospace applications. It should be remembered, however, that describing a substance as an ideal gas constitutes a model of the actual physical situation, and the limits of model validity must always be kept in mind. One of the other important features of an ideal gas is that its internal energy depends only upon its temperature. (For now, this can be regarded as another aspect of the model of actual systems that the ideal gas represents, but it can be shown that this is a consequence of the form of the equation of state.) Since depends only on ,

or

In the above equation we have indicated that can depend on . Like the internal energy, the enthalpy is also only dependent on for an ideal gas. (If a function of Therefore, , then, using the ideal gas equation of state, is also.)

is

and

If we are interested in finite changes of internal energy or enthalpy, we integrate,

and

Over small temperature changes (

), it is often assumed that

and

are constant. Furthermore, there are wide ranges over which specific heats do not vary greatly with respect to temperature, as shown in SB&VW Figure 5.11. It is thus often useful to treat them as constant. If so

These equations are useful in calculating internal energy or enthalpy differences, but it should be remembered that they hold only if the specific heats are constant. We can relate the specific heats of an ideal gas to its gas constant as follows. We write the first law in terms of internal energy,

and assume a quasi-static process so that we can also write it in terms of enthalpy, as in Section 2.3.4,

Equating the two first law expressions above, and assuming an ideal gas, we obtain

Combining terms,

Since

An expression that will appear often is the ratio of specific heats, which we will define as

Below we summarize the important results for all ideal gases, and give some values for specific types of ideal gases. 1. All ideal gases: 1. The specific heat at constant volume ( one kmol) is a function of only. 2. The specific heat at constant pressure ( one kmol) is a function of only. 3. A relation that connects the specific heats constant is for a unit mass or for a unit mass or , , and the gas for for

where the units depend on the mass considered. For a unit mass of gas, e.g., a kilogram, and would be the specific heats for one kilogram of gas and is as defined above. For one kmol of gas, the expression takes the form

where and have been used to denote the specific heats for one kmol of gas and is the universal gas constant.

4. The specific heat ratio, only and is greater than unity.

(or

), is a function of

2. An ideal gas with specific heats independent of temperature, and , is referred to as a perfect gas. For example, monatomic gases and diatomic gases at ordinary temperatures are considered perfect gases. To make this distinction the terminology "a perfect gas with constant specific heats" is used throughout the notes. In some textbooks perfect gases are sometimes also referred to as ideal gases, and to avoid confusion we use the stated terminology2.2. 3. Monatomic gases, such as He, Ne, Ar, and most metallic vapors: 1. (or ) is constant over a wide temperature range and is very [or , for one kmol].

nearly equal to 2. (or

) is constant over a wide temperature range and is very [or , for one kmol].

nearly equal to 3.

is constant over a wide temperature range and is very nearly equal to [ ]. ,O ,N , Air, NO, and

4. So-called permanent diatomic gases, namely H CO: 1. (or

) is nearly constant at ordinary temperatures, being

approximately [ , for one kmol], and increases slowly at higher temperatures.

2.

(or

) is nearly constant at ordinary temperatures, being

approximately [ , for one kmol], and increases slowly at higher temperatures. 3. is constant over a temperature range of roughly and is very nearly equal to temperature above this. [ to

]. It decreases with ,

5. Polyatomic gases and gases that are chemically active, such as CO NH , CH , and Freons:

The specific heats, and , and vary with the temperature, the variation being different for each gas. The general trend is that heavy molecular weight gases (i.e., more complex gas molecules than those listed in 2 or 3), have values of closer to unity than diatomic gases, which, as can be seen above, are closer to unity than monatomic gases. For example, values of below 1.2 are typical of Freons which have molecular weights of over one hundred.2.3 In general, for substances other than ideal gases, and depend on pressure as well as on temperature, and the above relations will not all apply. In this respect, the ideal gas is a very special model. In summary, the specific heats are thermodynamic properties and can be used even if the processes are not constant pressure or constant volume. The simple relations between changes in energy (or enthalpy) and temperature are a consequence of the behavior of an ideal gas, specifically the dependence of the energy and enthalpy on temperature only, and are not true for more complex substances.2.4

2.4.2 Reversible adiabatic processes for an ideal gas


From the first law, with , , and ,

(2..7)

Also, using the definition of enthalpy, (2..8)

The underlined terms are zero for an adiabatic process. Rewriting (2.7) and (2.8),

Combining the above two equations we obtain (2..9)

Equation (2.9) can be integrated between states 1 and 2 to give

For an ideal gas undergoing a reversible, adiabatic process, the relation between pressure and volume is thus:

We can substitute for or in the above result using the ideal gas law, or carry out the derivation slightly differently, to also show that

We will use the above equations to relate pressure and temperature to one another for quasi-static adiabatic processes (for instance, this type of process is our idealization of what happens in compressors and turbines).

Practice Questions 1. On a diagram for a closed-system sketch the thermodynamic paths to

that the system would follow if expanding from 2. 3. 4. 5.

by isothermal and quasi-static, adiabatic processes. For which process is the most work done by the system? For which process is there heat exchange? Is it added or removed? Is the final state of the system the same after each process? Derive expressions for the work done by the system for each process.

2.5 Control volume form of the system laws


[VWB&S, 6.1, 6.2] The thermodynamic laws (as well as Newton's laws) are for a system, a specific quantity of matter. More often, in propulsion and power problems, we are interested in what happens in a fixed volume, for example a rocket motor or a jet engine through which mass is flowing at a certain rate. We may also be interested in the rates of heat and work into and out of a system. For this reason, the control volume form of the system laws is of great importance. A schematic of the difference is shown in Figure 2.8. Rather than focus on a particle of mass which moves through the engine, it is more convenient to focus on the volume occupied by the engine. This requires us to use the control volume form of the thermodynamic laws, developed below.

Figure 2.8: Control volume and system for flow through a propulsion device

2.5.1 Conservation of mass


For the control volume shown, the rate of change of mass inside the volume is given by the difference between the mass flow rate in and the mass flow rate out. For a single flow coming in and a single flow coming out this is

If the mass inside the control volume changes with time it is because some mass is added or some is taken out. In the special case of a steady flow, therefore ,

Figure 2.9: A control volume used to track mass flows

2.5.2 Conservation of energy


The first law of thermodynamics can be written as a rate equation:

where

To derive the first law as a rate equation for a control volume we proceed as with the mass conservation equation. The physical idea is that any rate of change of energy in the control volume must be caused by the rates of energy flow into or out of the volume. The heat transfer and the work are already included and the only other contribution must be associated with the mass flow in and out, which

carries energy with it. Figure 2.10 shows two schematics of this idea. The desired form of the equation will be

[Simple]

[More

General] Figure 2.10: Schematic diagrams illustrating terms in the energy equation for a simple and a more general control volume The fluid that enters or leaves has an amount of energy per unit mass given by

where is the fluid velocity relative to some coordinate system, and we have neglected chemical energy. In addition, whenever fluid enters or leaves a control volume there is a work term associated with the entry or exit. We saw this in Section 2.3, example 1, and the present derivation is essentially an application of the ideas presented there. Flow exiting at station ``e'' must push back the surrounding fluid, doing work on it. Flow entering the volume at station ``i'' is pushed on by, and receives work from the surrounding air. The rate of flow work at exit is given by the product of the pressure times the exit area times the rate at

which the external flow is ``pushed back.'' The latter, however, is equal to the volume per unit mass times the rate of mass flow. Put another way, in a time the work done on the surroundings by the flow at the exit station is The net rate of flow work is

Including all possible energy flows (heat, shaft work, shear work, piston work, etc.), the first law can then be written as:

where includes the sign associated with the energy flow. If heat is added or work is done on the system then the sign is positive, if work or heat are extracted from the system then the sign is negative. NOTE: this is consistent with , where is the work done by the system on the environment, thus work is flowing out of the system. We can then combine the specific internal energy term, flow work term, , to make the enthalpy appear: , in and the specific

Thus, the first law can be written as:

For most of the applications in this course, there will be no shear work and no piston work. Hence, the first law for a control volume will be most often used as:

(2..10)

Note how our use of enthalpy has simplified the rate of work term. In writing the control volume form of the equation we have assumed only one entering and one leaving stream, but this could be generalized to any number of inlet and exit streams. In the special case of a steady-state flow,

Applying this to Equation 2.10 produces a form of the ``Steady Flow Energy Equation'' (SFEE), (2..11)

which has units of Joules per second. We could also divide by the mass flow to produce

which has units of Joules per second per kilogram. For problems of interest in aerospace applications the velocities are high and the term that is associated with changes in the elevation is small. From now on, we will neglect the unless explicitly stated. terms

Muddy Points What is shaft work? (MP 2.5) What distinguishes shaft work from other works? (MP 2.6) Definition of a control volume (MP 2.7)

2.5.3 Stagnation Temperature and Stagnation Enthalpy


Suppose that our steady flow control volume is a set of streamlines describing the flow up to the nose of a blunt object, as in Figure 2.11.

Figure 2.11: Streamlines and a stagnation region; a control volume can be drawn between the dashed streamlines and points 1 and 2 The streamlines are stationary in space, so there is no external work done on the fluid as it flows. If there is also no heat transferred to the flow (adiabatic), then the steady flow energy equation becomes

The quantity that is conserved is defined as the stagnation temperature,

or

where is the Mach number2.5. The stagnation temperature is the temperature that the fluid would reach if it were brought to zero speed by a steady adiabatic process with no external work. Note that for any steady, adiabatic flow with no external work, the stagnation temperature is constant. It is also convenient to define the stagnation enthalpy,

which allows us to write the Steady Flow Energy Equation in a simpler form as

Note that for a quasi-static adiabatic process

so we can write

and define the relationship between stagnation pressure and static pressure as

where, the stagnation pressure is the pressure that the fluid would reach if it were brought to zero speed, via a steady, adiabatic, quasi-static process with no external work.

2.5.3.1 Frame dependence of stagnation quantities

An area of common confusion is the frame dependence of stagnation quantities. The stagnation temperature and stagnation pressure are the conditions the fluid would reach if it were brought to zero speed relative to some reference frame, via a steady adiabatic process with no external work (for stagnation temperature) or a steady, adiabatic, reversible process with no external work (for stagnation pressure). Depending on the speed of the reference frame the stagnation quantities will take on different values. For example, consider a high speed reentry vehicle traveling through the still atmosphere, which is at temperature, . Let's place our reference frame on the vehicle and stagnate a fluid particle on the nose of the vehicle (carrying it along with the vehicle and thus essentially giving it kinetic energy). The stagnation temperature of the air in the vehicle frame is

where is the vehicle speed. The temperature the skin reaches (to first approximation) is the stagnation temperature and depends on the speed of the vehicle. Since re-entry vehicles travel fast, the skin temperature is much hotter than the atmospheric temperature. The atmospheric temperature, , is not frame dependent, but the stagnation temperature, , is.

The confusion comes about because is usually referred to as the static temperature. In common language this has a similar meaning as ``stagnation,'' but in fluid mechanics and thermodynamics static is used to label the thermodynamic properties of the gas ( dependent. , , etc.), and these are not frame

Thus in our re-entry vehicle example, looking at the still atmosphere from the vehicle frame we see a stagnation temperature hotter than the atmospheric (static) temperature. If we look at the same still atmosphere from a stationary frame, the stagnation temperature is the same as the static temperature.

2.5.3.2 Example
For the case shown below, a jet engine is sitting motionless on the ground prior to take-off. Air is entrained into the engine by the compressor. The inlet can be assumed to be frictionless and adiabatic.

Figure 2.12: A stationary gas turbine drawing air in from the atmosphere Considering the state of the gas within the inlet, prior to passage into the compressor, as state (1), and working in the reference frame of the motionless airplane: 1. Is greater than, less than, or equal to ?

The stagnation temperature of the atmosphere, , is equal to since it is moving the same speed as the reference frame (the motionless airplane). The steady flow energy equation tells us that if there is no heat or shaft work (the case for our adiabatic inlet) the stagnation enthalpy (and thus stagnation temperature for constant ) remains unchanged. Thus

2. Is

greater than, less than, or equal to

If then since the flow is moving at station 1 and therefore some of the total energy is composed of kinetic energy (at the expense of internal energy, thus lowering 3. Is greater than, less than, or equal to ) ?

Equal, by the same argument as 1. 4. Is greater than, less than, or equal to ?

Less than, by the same argument as 2.

2.5.3.3 Steady Flow Energy Equation in terms of Stagnation Enthalpy

The form of the ``Steady Flow Energy Equation'' (SFEE) that we will most commonly use is Equation 2.11 written in terms of stagnation quantities, and neglecting chemical and potential energies,

The steady flow energy equation finds much use in the analysis of power and propulsion devices and other fluid machinery. Note the prominent role of enthalpy.

Muddy Points What is the difference between enthalpy and stagnation enthalpy? (MP 2.8)

2.5.4 Example Applications of the Steady Flow Energy Equation


[VW, S& B: 6.4]

2.5.4.1 Tank Filling


Using what we have just learned we can attack the tank filling problem solved in Section 2.3.3 from an alternate point of view using the control volume form of the first law. In this problem the shaft work is zero, and the heat transfer, kinetic energy changes, and potential energy changes are neglected. In addition there is no exit mass flow.

Figure 2.13: A control volume approach to the tank filling problem The control volume form of the first law is therefore

The equation of mass conservation is

Combining we have

Integrating from the initial time to the final time (the incoming enthalpy is constant) and using gives the result as before.

2.5.4.2 Flow through a rocket nozzle


A liquid bi-propellant rocket consists of a thrust chamber and nozzle and some means for forcing the liquid propellants into the chamber where they react, converting chemical energy to thermal energy.

Figure 2.14: Flow through a rocket nozzle Once the rocket is operating we can assume that all of the flow processes are steady, so it is appropriate to use the steady flow energy equation. Also, for now we will assume that the gas behaves as a perfect gas with constant specific heats, though in general this is a poor approximation. There is no external work, and we assume that the flow is adiabatic. We define our control volume as going between location , in the chamber, and location , at the exit, and then write the First Law as

or

Therefore

If we assume quasi-static, adiabatic expansion then

so

and

, the conditions in the combustion chamber, are set by propellants, and

is the external static pressure.

2.5.4.3 Power to drive a gas turbine compressor


Consider for example the PW4084 pictured in Figure 2.15. The engine is designed to produce about 84,000 lbs of thrust at takeoff. The engine is a twospool design. The fan and low pressure compressor are driven by the low pressure turbine. The high pressure compressor is driven by the high pressure turbine. We wish to find the total shaft work required to drive the compression system.

Figure 2.15: The Pratt and Whitney 4084 (drawing courtesy of Pratt and Whitney)

We define our control volume to encompass the compression system, from the front of the fan to the back of the fan and high pressure compressor, with the shaft cutting through the back side of the control volume. Heat transfer from the gas streams is negligible, so we write the First Law (steady flow energy equation) as:

For this problem we must consider two streams, the fan stream, stream, :

, and the core

We obtain the temperature change by assuming that the compression process is quasi-static and adiabatic,

then

Substituting these values into the expression for the first law above, along with estimates of , we obtain

Note that . If a car engine has then the power needed to drive compressor is equivalent to 1,110 automobile engines. All of this power is generated by the low pressure and high pressure turbines.

2.6 Muddiest Points on Chapter 2


MP 2..1 What are the conventions for work and heat in the first law?

Heat is positive if it is given to the system. Work is positive if it is done by the system. MP 2..2 When does ?

We deal with changes in energy. When the changes in the other types of energy (kinetic, potential, strain, etc.) can be neglected compared to the changes in thermal energy, then it is a good approximation to use as representing the total energy change. MP 2..3 When is enthalpy the same in initial and final states? Initial and final stagnation enthalpy is the same if the flow is steady and if there is no net shaft work plus heat transfer. If the change in kinetic energy is negligible, the initial and final enthalpy is the same. The tank problem is unsteady so the initial and final enthalpies are not the same. See the discussion of the steady flow energy equation in notes, Section 2.5. MP 2..4 In the filling of a tank, why (physically) is the final temperature in the tank higher than the initial temperature? Work is done on the system, which in this problem is the mass of gas that is pushed into the tank. MP 2..5 What is shaft work? I am not sure how best to answer, but it appears that the difficulty people are having might be associated with being able to know when one can say that shaft work occurs. There are several features of a process that produces (or absorbs) shaft work. First of all the view taken of the process is one of control volume, rather than control mass (see the discussion of control volumes in Chapter I or in IAW). Second, there needs to be a shaft or equivalent device (a moving belt, a row of blades) that can be identified as the work carrier. Third, the shaft work is work over and above the ``flow work'' that is done by (or received by) the streams that exit and enter the control volume. MP 2..6 What distinguishes shaft work from other works? The term shaft work arises in using a control volume approach. As we have defined it, ``shaft work'' is all work over and above work associated with the ``flow work'' (the work done by pressure forces). Generally this means work done by rotating machinery, which is carried by a shaft from the control volume to the outside world. There could also be work over and above the pressure force work done by shear stresses at the boundaries of the control volume, but this is seldom important if the control boundary is normal to the flow direction.

If we consider a system (a mass of fixed identity, say a blob of gas) flowing through some device, neglecting the effects of raising or lowering the blob the only mode of work would be the work to compress the blob. This would be true even if the blob were flowing through a turbine or compressor. (In doing this we are focusing on the same material as it undergoes the unsteady compression or expansion processes in the device, rather than looking at a control volume, through which mass passes.) The question about shaft work and non shaft work has been asked several times. I am not sure how best to answer, but it appears that the difficulty people are having might be associated with being able to know when one can say that shaft work occurs. There are several features of a process that produces (or absorbs) shaft work. First of all, the view taken of the process is one of control volume, rather than control mass (see the discussion of control volumes in Section 2.5 or in IAW). Second, there needs to be a shaft or equivalent device (a moving belt, a row of blades) that can be identified as the work carrier. Third, the shaft work is work over and above the flow work that is done by (or received by) the streams that exit and enter the control volume. MP 2..7 Definition of a control volume. A control volume is an enclosure that separates a quantity of matter from the surroundings or environment. The enclosure does not necessarily have to consist of a solid boundary like the walls of a vessel. It is only necessary that the enclosure forms a closed surface and that its properties are defined everywhere. An enclosure may transmit heat or be a heat insulator. It may be deformable and thus capable of transmitting work to the system. It may also be capable of transmitting mass. MP 2..8 What is the difference between enthalpy and stagnation enthalpy?

The enthalpy of a gas is defined as , and represents both the internal energy of that state and the flow work done on the gas to get it at that pressure and density. The stagnation enthalpy of a gas is defined as and accounts for both the enthalpy and the kinetic energy of the gas at that state.

3. The First Law Applied to Engineering Cycles


[VW, S & B: Chapter 9, 11.8, 11.9, 11.10, 11.11, 11.12, 11.13, 11.14]

This chapter is devoted to describing the fundamentals of how various heat engines work. A heat engine is a device that uses heat to produce work, or uses work to move around heat. Refrigerators, internal combustion (automobile) engines, and jet engines are all types of heat engines. We will model these heat engines as thermodynamic cycles and apply the First Law of Thermodynamics to estimate thermal efficiency and work output as a function of pressures and temperatures at various points in the cycle. This is called ideal cycle analysis. The estimates we obtain from the analysis represent the best achievable performance that may be obtained from a heat engine. In reality, the performance of these systems will be somewhat less than the estimates obtained from ideal cycle analysis - you will learn how to make more realistic estimates later. Our analyses will use the ``air-standard cycle,'' which is an approximation to actual cycle behavior. Specifically, we make the following simplifications:

Air is the working fluid (the presence of fuel and combustion products is neglected)3.1, Combustion is represented by heat transfer from an external heat source, The cycle is `completed' by heat transfer to the surroundings, All processes are internally reversible (described more fully in Chapter 5), and Air is a perfect gas with constant specific heats.

3.1 Some Properties of Engineering Cycles; Work and Efficiency


As preparation for our discussion of cycles (and as a foreshadowing of the second law), we examine two types of processes that concern interactions between heat and work. The first of these represents the conversion of work into heat. The second, which is much more useful, concerns the conversion of heat into work. The question we will pose is how efficient can this conversion be in the two cases.

Figure 3.1: Examples of the conversion of work into heat

Three examples of the first process are given in Figure 3.1. The first is the pulling of a block on a rough horizontal surface by a force which moves through some distance. Friction resists the pulling. After the force has moved through the distance, it is removed. The block then has no kinetic energy and the same potential energy it had when the force started to act. If we measured the temperature of the block and the surface we would find that it was higher than when we started. (High temperatures can be reached if the velocities of pulling are high; this is the basis of inertia welding.) The work done to move the block has been converted totally to heat. The second example concerns the stirring of a viscous liquid. There is work associated with the torque exerted on the shaft turning through an angle. When the stirring stops, the fluid comes to rest and there is (again) no change in kinetic or potential energy from the initial state. The fluid and the paddle wheels will be found to be hotter than when we started, however. The final example is the passage of a current through a resistance. This is a case of electrical work being converted to heat, indeed it models operation of an electrical heater. All the examples in Figure 3.1 have 100% conversion of work into heat. This 100% conversion could go on without limit as long as work were supplied. Is this true for the conversion of heat into work? To answer the last question, we need to have some basis for judging whether work is done in a given process. One way to do this is to ask whether we can construct a way that the process could result in the raising of a weight in a gravitational field. If so, we can say ``Work has been done.'' It may sometimes be difficult to make the link between a complicated thermodynamic process and the simple raising of a weight, but this is a rigorous test for the existence of work. One example of a process in which heat is converted to work is the isothermal (constant temperature) expansion of an ideal gas, as sketched in Figure 3.2. The system is the gas inside the chamber. As the gas expands, the piston does work on some external device. For an ideal gas, the internal energy is a function of temperature only, so that if the temperature is constant for some process the internal energy change is zero. To keep the temperature constant during the expansion, heat must be supplied. Because the first law takes the form

. This is a process that has 100% conversion of heat into work.

Figure 3.2: Isothermal expansion The work exerted by the system is given by

where 1 and 2 denote the two states at the beginning and end of the process. The equation of state for an ideal gas is with the number of moles of the gas contained in the chamber. Using the equation of state, the expression for work can be written as (3..1)

For an isothermal process, , so that . The work can be written in terms of the pressures at the beginning and end as (3..2)

The lowest pressure to which we can expand and still receive work from the system is atmospheric pressure. Below this, we would have to do work on the system to pull the piston out further. There is thus a bound on the amount of work that can be obtained in the isothermal expansion; we cannot continue indefinitely. For a power or propulsion system, however, we would like a source of continuous power, in other words a device that would give power or propulsion as long as fuel was added to it. To do this, we need a series of processes where the system does not progress through a one-way transition from an initial state to a different final state, but rather cycles back to the initial state. What is looked for is in fact a thermodynamic cycle for the system.

We define several quantities for a cycle:


is the heat absorbed by the system. is the heat rejected by the system. is the net work done by the system.

The cycle returns to its initial state, so the overall energy change, , is zero. The net work done by the system is related to the magnitudes of the heat absorbed and the heat rejected by

The thermal efficiency of the cycle is the ratio of the work done to the heat absorbed. (Efficiencies are often usefully portrayed as ``What you get'' versus ``What you pay for.'' Here what we get is work and what we pay for is heat, or rather the fuel that generates the heat.) In terms of the heat absorbed and rejected, the thermal efficiency is (3..3)

The thermal efficiency can only be 100% (complete conversion of heat into work) if ; a basic question is what is the maximum thermal efficiency for any arbitrary cycle? We examine this for several cases, including the Carnot cycle and the Brayton (or Joule) cycle, which is a model for the power cycle in a jet engine.

3.2 Generalized Representation of Thermodynamic Cycles


[VW, S & B: 6.1] Before we examine individual heat engines, note that all heat engines can be represented generally as a transfer of heat from a high temperature reservoir to a device, which does work on the surroundings, followed by a rejection of heat from that device to a low temperature reservoir.

Figure 3.3: A generalized heat engine

3.3 The Carnot Cycle


A Carnot cycle is shown in Figure 3.4. It has four processes. There are two adiabatic reversible legs and two isothermal reversible legs. We can construct a Carnot cycle with many different systems, but the concepts can be shown using a familiar working fluid, the ideal gas. The system can be regarded as a chamber enclosed by a piston and filled with this ideal gas.

Figure 3.4: Carnot cycle -- thermodynamic diagram on left and schematic of the different stages in the cycle for a system composed of an ideal gas on the right The four processes in the Carnot cycle are: 1. The system is at temperature at state . It is brought in contact with a heat reservoir, which is just a liquid or solid mass of large enough extent such that its temperature does not change appreciably when some amount of heat is transferred to the system. In other words, the heat reservoir is a constant temperature source (or receiver) of heat. The system then undergoes an isothermal expansion from to , with heat absorbed .

2. At state , the system is thermally insulated (removed from contact with the heat reservoir) and then let expand to . During this expansion the temperature decreases to cycle, 3. At state . The heat exchanged during this part of the

) the system is brought in contact with a heat reservoir at

temperature . It is then compressed to state , rejecting heat in the process. 4. Finally, the system is compressed adiabatically back to the initial state . The heat exchange .

The thermal efficiency of the cycle is given by the definition (3..4)

In this equation, there is a sign convention implied. The quantities , as defined are the magnitudes of the heat absorbed and rejected. The quantities , on the other hand are defined with reference to heat received by the system. In this example, the former is negative and the latter is positive. The heat absorbed and rejected by the system takes place during isothermal processes and we already know what their values are from Eq. (3.1):

The efficiency can now be written in terms of the volumes at the different states as (3..5)

The path from states to and from to are both adiabatic and reversible. For a reversible adiabatic process we know that . Using the ideal

gas equation of state, we have . Along the curve ,

. Along curve -

, therefore,

. Thus,

Comparing the expression for thermal efficiency Eq. (3.4) with Eq. (3.5) shows two consequences. First, the heats received and rejected are related to the temperatures of the isothermal parts of the cycle by (3..6)

Second, the efficiency of a Carnot cycle is given compactly by (3..7)

The efficiency can be 100% only if the temperature at which the heat is rejected is zero. The heat and work transfers to and from the system are shown schematically in Figure 3.5.

Figure 3.5: Work and heat transfers in a Carnot cycle between two heat reservoirs

Muddy Points

Since

, looking at the

graph, does that mean the farther

apart the , isotherms are, the greater efficiency? And that if they were very close, it would be very inefficient? (MP 3.2) In the Carnot cycle, why are we only dealing with volume changes and not pressure changes on the adiabats and isotherms? (MP 3.3) Is there a physical application for the Carnot cycle? Can we design a Carnot engine for a propulsion device? (MP 3.4) How do we know which cycles to use as models for real processes? (MP 3.5)

3.4 Refrigerators and Heat Pumps


The Carnot cycle has been used for power, but we can also run it in reverse. If so, there is now net work into the system and net heat out of the system. There will be a quantity of heat rejected at the higher temperature and a quantity of

heat absorbed at the lower temperature. The former of these is negative according to our convention and the latter is positive. The result is that work is done on the system, heat is extracted from a low temperature source and rejected to a high temperature source. The words ``low'' and ``high'' are relative and the low temperature source might be a crowded classroom on a hot day, with the heat extraction being used to cool the room. The cycle and the heat and work transfers are indicated in Figure 3.6. In this mode of operation the cycle works as a refrigerator or heat pump. ``What we pay for'' is the work, and ``what we get'' is the amount of heat extracted. A metric for devices of this type is the coefficient of performance, defined as

Figure 3.6: Operation of a Carnot refrigerator For a Carnot cycle we know the ratios of heat in to heat out when the cycle is run forward and, since the cycle is reversible, these ratios are the same when the cycle is run in reverse. The coefficient of performance is thus given in terms of the absolute temperatures as

This can be much larger than unity. The Carnot cycles that have been drawn are based on ideal gas behavior. For different working media, however, they will look different. We will see an example when we discuss two-phase situations. What is the same whatever the medium is the efficiency for all Carnot cycles operating between the same two temperatures.

3.4.0.1 Refrigerator Hardware


Typically the thermodynamic system in a refrigerator analysis will be a working fluid, a refrigerant, that circulates around a loop, as shown in Figure 3.7. The internal energy (and temperature) of the refrigerant is alternately raised and lowered by the devices in the loop. The working fluid is colder than the refrigerator air at one point and hotter than the surroundings at another point. Thus heat will flow in the appropriate direction, as shown by the two arrows in the heat exchangers.

Figure 3.7: Schematic of a domestic refrigerator Starting in the upper right hand corner of the diagram, we describe the process in more detail. First the refrigerant passes through a small turbine or through an expansion valve. In these devices, work is done by the refrigerant so its internal energy is lowered to a point where the temperature of the refrigerant is lower than that of the air in the refrigerator. A heat exchanger is used to transfer energy from the inside of the refrigerator to the cold refrigerant. This lowers the internal energy of the inside and raises the internal energy of the refrigerant. Then a pump or compressor is used to do work on the refrigerant, adding additional energy to it and thus further raising its internal energy. Electrical energy is used to drive the pump or compressor. The internal energy of the refrigerant is raised to a point where its temperature is hotter than the temperature of the surroundings. The refrigerant is then passed through a heat exchanger (often coils at the back of the refrigerator) so that energy is transferred from the refrigerant to the surroundings. As a result, the internal energy of the refrigerant is reduced and the internal energy of the surroundings is increased. It is at this point where the internal energy of the contents of the refrigerator and the energy used to drive the compressor or pump are transferred to the surroundings. The refrigerant then continues on to the turbine or expansion valve, repeating the cycle.

3.5 The Internal combustion engine (Otto Cycle)


[VW, S & B: 9.13] The Otto cycle is a set of processes used by spark ignition internal combustion engines (2-stroke or 4-stroke cycles). These engines a) ingest a mixture of fuel and air, b) compress it, c) cause it to react, thus effectively adding heat through converting chemical energy into thermal energy, d) expand the combustion

products, and then e) eject the combustion products and replace them with a new charge of fuel and air. The different processes are shown in Figure 3.8: 1. Intake stroke, gasoline vapor and air drawn into engine ( 2. Compression stroke, , increase ( ). 3. Combustion (spark), short time, essentially constant volume ( ). ).

Model: heat absorbed from a series of reservoirs at temperatures to . 4. Power stroke: expansion ( ). 5. Valve exhaust: valve opens, gas escapes. 6. ( ) Model: rejection of heat to series of reservoirs at temperatures to . 7. Exhaust stroke, piston pushes remaining combustion products out of chamber ( ). We model the processes as all acting on a fixed mass of air contained in a piston-cylinder arrangement, as shown in Figure 3.10.

Figure 3.8: The ideal Otto cycle

Figure 3.9: Sketch of an actual Otto cycle

Figure 3.10: Piston and valves in a four-stroke internal combustion engine The actual cycle does not have the sharp transitions between the different processes that the ideal cycle has, and might be as sketched in Figure 3.9.

3.5.1 Efficiency of an ideal Otto cycle


The starting point is the general expression for the thermal efficiency of a cycle:

The convention, as previously, is that heat exchange is positive if heat is flowing into the system or engine, so is negative. The heat absorbed occurs during combustion when the spark occurs, roughly at constant volume. The heat absorbed can be related to the temperature change from state 2 to state 3 as:

The heat rejected is given by (for a perfect gas with constant specific heats) Substituting the expressions for the heat absorbed and rejected in the expression for thermal efficiency yields

We can simplify the above expression using the fact that the processes from 1 to 2 and from 3 to 4 are isentropic:

The quantity is called the compression ratio. In terms of compression ratio, the efficiency of an ideal Otto cycle is:

Figure 3.11: Ideal Otto cycle thermal efficiency The ideal Otto cycle efficiency is shown as a function of the compression ratio in Figure 3.11. As the compression ratio, , increases, increases, but so does

. If is too high, the mixture will ignite without a spark (at the wrong location in the cycle).

3.5.2 Engine work, rate of work per unit enthalpy flux


The non-dimensional ratio of work done (the power) to the enthalpy flux through the engine is given by

There is often a desire to increase this quantity, because it means a smaller engine for the same power. The heat input is given by

where

is the heat of reaction, i.e. the chemical energy liberated per unit mass of fuel, is the fuel mass flow rate.

The non-dimensional power is

The quantities in this equation, evaluated at stoichiometric conditions are:

so

Muddy Points How is calculated? (MP 3.6)

What are ``stoichiometric conditions?'' (MP 3.7)

3.6 Diesel Cycle


The Diesel cycle is a compression ignition (rather than spark ignition) engine. Fuel is sprayed into the cylinder at (high pressure) when the compression is complete, and there is ignition without a spark. An idealized Diesel engine cycle is shown in Figure 3.12.

Figure 3.12: The ideal Diesel cycle The thermal efficiency is given by:

This cycle can operate with a higher compression ratio than the Otto cycle because only air is compressed and there is no risk of auto-ignition of the fuel. Although for a given compression ratio the Otto cycle has higher efficiency, because the Diesel engine can be operated to higher compression ratio, the engine can actually have higher efficiency than an Otto cycle when both are operated at compression ratios that might be achieved in practice.

Muddy Points

When and where do we use ever ? (MP 3.8)

and

? Some definitions use

. Is it

Explanation of the above comparison between Diesel and Otto. (MP 3.9)

3.7 Brayton Cycle


[VW, S & B: 9.8-9.9, 9.12] The Brayton cycle (or Joule cycle) represents the operation of a gas turbine engine. The cycle consists of four processes, as shown in Figure 3.13 alongside a sketch of an engine:

a - b Adiabatic, quasi-static (or reversible) compression in the inlet and compressor; b - c Constant pressure fuel combustion (idealized as constant pressure heat addition); c - d Adiabatic, quasi-static (or reversible) expansion in the turbine and exhaust nozzle, with which we 1. take some work out of the air and use it to drive the compressor, and 2. take the remaining work out and use it to accelerate fluid for jet propulsion, or to turn a generator for electrical power generation; d - a Cool the air at constant pressure back to its initial condition.

Figure 3.13: Sketch of the jet engine components and corresponding thermodynamic states The components of a Brayton cycle device for jet propulsion are shown in Figure 3.14. We will typically represent these components schematically, as in Figure 3.15. In practice, real Brayton cycles take one of two forms. Figure 3.16(a) shows an ``open'' cycle, where the working fluid enters and then exits the device.

This is the way a jet propulsion cycle works. Figure 3.16(b) shows the alternative, a closed cycle, which recirculates the working fluid. Closed cycles are used, for example, in space power generation.

Figure 3.14: Schematics of typical military gas turbine engines. Top: turbojet with afterburning, bottom: GE F404 low bypass ratio turbofan with afterburning (Hill and Peterson, 1992).

Figure 3.15: Thermodynamic model of gas turbine engine cycle for power generation

[Open cycle operation]

[Closed cycle operation]

Figure 3.16: Options for operating Brayton cycle gas turbine engines

Muddy Points Would it be practical to run a Brayton cycle in reverse and use it as refrigerator? (MP 3.10)

3.7.1 Work and Efficiency


The objective now is to find the work done, the heat absorbed, and the thermal efficiency of the cycle. Tracing the path shown around the cycle from - - and back to , the first law gives (writing the equation in terms of a unit mass),

Here is zero because is a function of state, and any cycle returns the system to its starting state3.2. The net work done is therefore

where , are defined as heat received by the system ( thus need to evaluate the heat transferred in processes -

is negative). We and - .

For a constant pressure, quasi-static process the heat exchange per unit mass is

We can see this by writing the first law in terms of enthalpy (see Section 2.3.4) or by remembering the definition of .

The heat exchange can be expressed in terms of enthalpy differences between the relevant states. Treating the working fluid as a perfect gas with constant specific heats, for the heat addition from the combustor,

The heat rejected is, similarly, The net work per unit mass is given by

The thermal efficiency of the Brayton cycle can now be expressed in terms of the temperatures: (3..8)

To proceed further, we need to examine the relationships between the different temperatures. We know that points and are on a constant pressure process as are points and , and are adiabatic and reversible, so ; . The other two legs of the cycle

Therefore , or, finally, . Using this relation in the expression for thermal efficiency, Eq. (3.8) yields an expression for the thermal efficiency of a Brayton cycle: (3..9)

The temperature ratio across the compressor, . In terms of compressor temperature ratio, and using the relation for an adiabatic reversible process we can write the efficiency in terms of the compressor (and cycle) pressure ratio, which is the parameter commonly used: (3..10)

Figure 3.17: Gas turbine engine pressures and temperatures Figure 3.17 shows pressures and temperatures through a gas turbine engine (the PW4000, which powers the 747 and the 767).

Figure 3.18: Gas turbine engine pressure ratio trends (Janes Aeroengines, 1998)

Figure 3.19: Trend of Brayton cycle thermal efficiency with compressor pressure

ratio Equation (3.10) says that for a high cycle efficiency, the pressure ratio of the cycle should be increased. This trend is plotted in Figure 3.19. Figure 3.18 shows the history of aircraft engine pressure ratio versus entry into service, and it can be seen that there has been a large increase in cycle pressure ratio. The thermodynamic concepts apply to the behavior of real aerospace devices!

Muddy Points When flow is accelerated in a nozzle, doesn't that reduce the internal energy of the flow and therefore the enthalpy? (MP 3.11) Why do we say the combustion in a gas turbine engine is constant pressure? (MP 3.12) Why is the Brayton cycle less efficient than the Carnot cycle? (MP 3.13) If the gas undergoes constant pressure cooling in the exhaust outside the engine, is that still within the system boundary? (MP 3.14) Does it matter what labels we put on the corners of the cycle or not? (MP 3.15) Is the work done in the compressor always equal to the work done in the turbine plus work out (for a Brayton cyle)? (MP 3.16)

3.7.2 Gas Turbine Technology and Thermodynamics


The turbine entry temperature, , is fixed by materials technology and cost. (If the temperature is too high, the blades fail.) Figures 3.20 and 3.21 show the progression of the turbine entry temperatures in aeroengines. Figure 3.20 is from Rolls Royce and Figure 3.21 is from Pratt & Whitney. Note the relation between the gas temperature coming into the turbine blades and the blade melting temperature.

Figure 3.20: Rolls-Royce high temperature technology

Figure 3.21: Turbine blade cooling technology [Pratt & Whitney] For a given level of turbine technology (in other words given maximum temperature) a design question is what should the compressor be? What criterion should be used to decide this? Maximum thermal efficiency? Maximum work? We examine this issue below.

Figure 3.22: Efficiency and work of two Brayton cycle engines

The problem is posed in Figure 3.22, which shows two Brayton cycles. For maximum efficiency we would like as high as possible. This means that the compressor exit temperature approaches the turbine entry temperature. The net work will be less than the heat received; as approaches zero and so does the net work. the heat received

The net work in the cycle can also be expressed as , evaluated in traversing the cycle. This is the area enclosed by the curves, which is seen to approach zero as .

The conclusion from either of these arguments is that a cycle designed for maximum thermal efficiency is not very useful in that the work (power) we get out of it is zero. A more useful criterion is that of maximum work per unit mass (maximum power per unit mass flow). This leads to compact propulsion devices. The work per unit mass is given by:

where is the maximum turbine inlet temperature (a design constraint) and is atmospheric temperature. The design variable is the compressor exit temperature, , and to find the maximum as this is varied, we differentiate the :

expression for work with respect to

The first and the fourth terms on the right hand side of the above equation are both zero (the turbine entry temperature is fixed, as is the atmospheric temperature). The maximum work occurs where the derivative of work with respect to is zero: (3..11)

To use Eq. (3.11), we need to relate

and

. We know that

Hence,

Plugging this expression for the derivative into Eq. (3.11) gives the compressor exit temperature for maximum work as ratio, . In terms of temperature

The condition for maximum work in a Brayton cycle is different than that for maximum efficiency. The role of the temperature ratio can be seen if we examine the work per unit mass which is delivered at this condition:

Ratioing all temperatures to the engine inlet temperature,

To find the power the engine can produce, we need to multiply the work per unit mass by the mass flow rate:

(3..12)

The trend of work output vs. compressor pressure ratio, for different temperature ratios , is shown in Figure 3.23.

Figure 3.23: Trend of cycle work with compressor pressure ratio, for different temperature ratios

[Gas turbine engine core]

[Core power vs. turbine

entry temperature] Figure 3.24: Aeroengine core power [Koff/Meese, 1995] Figure 3.24 shows the expression for power of an ideal cycle compared with data from actual jet engines. Figure 3.24(a) shows the gas turbine engine layout including the core (compressor, burner, and turbine). Figure 3.24(b) shows the core power for a number of different engines as a function of the turbine rotor entry temperature. The equation in the figure for horsepower (HP) is the same as that which we just derived, except for the conversion factors. The analysis not only shows the qualitative trend very well but captures much of the quantitative behavior too. A final comment (for this section) on Brayton cycles concerns the value of the thermal efficiency. The Brayton cycle thermal efficiency contains the ratio of the compressor exit temperature to atmospheric temperature, so that the ratio is not based on the highest temperature in the cycle, as the Carnot efficiency is. For a given maximum cycle temperature, the Brayton cycle is therefore less efficient than a Carnot cycle.

Muddy Points What are the units of in ? (MP 3.17)

Question about the assumptions made in the Brayton cycle for maximum efficiency and maximum work (MP 3.18) You said that for a gas turbine engine modeled as a Brayton cycle the work done is , where is the heat added and is the heat rejected. Does this suggest that the work that you get out of the engine doesn't depend on how good your compressor and turbine are? since the compression and expansion were modeled as adiabatic. (MP 3.19)

3.7.3 Brayton Cycle for Jet Propulsion: the Ideal Ramjet


A schematic of a ramjet is given in Figure 3.25.

Figure 3.25: Ideal ramjet [J. L. Kerrebrock, Aircraft Engines and Gas Turbines] In the ramjet there are ``no moving parts.'' The processes that occur in this propulsion device are:

: Isentropic diffusion (slowing down) and compression, with a decrease in Mach number, . : Constant pressure combustion. : Isentropic expansion through the nozzle.

The ramjet thermodynamic cycle efficiency can be written in terms of flight Mach number, , as follows:

and

so

See also Section 11.6.3 for other figures of merit.

Muddy Points Why don't we like the numbers 1 and 2 for the stations? Why do we go 0-3? (MP 3.20) For the Brayton cycle efficiency, why does ? (MP 3.21)

3.8 Muddiest points on Chapter 3


MP 3..1 How can we idealize fuel addition as heat addition? The validity of an approximation rests on what the answer is going to be used for. We are seeking basically only one item concerning combustor exit conditions, namely the exit temperature or the exit enthalpy. The final state is independent of how we add the heat, and depends only on whether we add the heat. If it is done from an electrical heater or from combustion, and if we neglect the change in the constitution of the gas due to the combustion products (most of the gas is nitrogen) the enthalpy rise is the same no matter how the temperature rise is achieved.

MP 3..2 Since

, looking at the

graph, does that mean

the farther apart the , isotherms are, the greater the efficiency? And that if they were very close, it would be very inefficient? This is correct. However, there is a limit on the maximum achievable efficiency. We cannot convert the absorbed heat into 100% work, that is, we always must reject some amount of heat. The amount of heat we must reject is

Thus for given values of

and

depends only on the temperature of the

cold reservoir , which is limited by the temperatures naturally available to us. These temperatures are all well above absolute zero, and there are no means to reduce to negligible values. The consequence of this is that the Carnot cycle only if , which is not possible).

efficiency cannot approach one (

MP 3..3 In the Carnot cycle, why are we only dealing with volume changes and not pressure changes on the adiabats and isotherms? We are not neglecting the pressure terms and we are also dealing with pressure changes. On the adiabats we know that (adiabatic process), so that for and, using for an ideal gas,

reversible processes we can write the first law as enthalpy, also as we can write the ratio of . With as and

By arranging terms we obtain

For a process we can integrate from 1 to 2 and get , or . This relation shows how pressure and volume changes are related to one another during an adiabatic reversible process. During an isothermal process, the temperature stays constant. Using the equation of state for an ideal gas, , we find that on an isotherm. Again, this relation tells us how pressure changes are related to volume changes during an isothermal process. Note that in the - diagram, adiabats ( ) are steeper curves than isotherms ( ). MP 3..4 Is there a physical application for the Carnot cycle? Can we design a Carnot engine for a propulsion device? We will see that Carnot cycles are the best we can do in terms of efficiency. A constant temperature heat transfer process is, however, difficult to attain in practice for devices in which high rates of power are required. The main role of the Carnot engine is therefore as a standard against which all other cycles are compared and which shows us the direction in which design of efficient cycles should go. MP 3..5 How do we know which cycles to use as models for real processes? We have discussed this briefly for the Brayton cycle, in that we looked at the approximation that was made in saying heat addition occurred at constant

pressure. You can also see that the Carnot cycle is not a good descriptor of a gas turbine engine! We will look further at this general point, not only for the Brayton cycle, but also for the Rankine cycle and for some internal combustion engine cycles. I will try to make clear what are the approximations and why the cycle under study is being used as a model. MP 3..6 How is calculated?

For now, we rely on tabulated values. In the lectures accompanying Chapter 15 of the notes, we will see how one can calculate the heat, given reaction. MP 3..7 What are ``stoichiometric conditions?'' Stoichiometric conditions are those in which the proportions of fuel and air are such that there is not an excess of either one in the combustion reaction -- all the fuel is burned, and all the air (oxidizer) is used up in the reaction. See Chapter 15. MP 3..8 When and where do we use . Is it ever ? and are derived in the notes in and ? Some definitions use , liberated in a

The answer is no. The definitions of Section 2.4.

is the specific heat at constant pressure and for an ideal gas is the specific heat at constant volume

always holds. Similarly

and for an ideal gas always holds. A discussion of this is also given in the notes in Section 2.4. If you think about how you would measure the specific heat for a certain known change of state you could do the following experiments. For a process during which heat is transferred (reversibly) and the volume stays constant (e.g. a rigid, closed container filled with a substance, or the heat transfer in an Otto engine during combustion -- the piston is near the top-deadcenter and the volume is approximately constant for the heat transfer) the first

law is since . Using the definition the specific heat at constant volume

we obtain for

where both the heat transferred measured.

and the temperature difference

can be

Similarly we can do an experiment involving a process where the pressure is kept constant during the reversible heat transfer (e.g. a rigid container filled with a substance that is closed by a lid with a certain weight, or the heat transfer in a jet engine combustor where the pressure is approximately constant during heat addition). The first law can be written in terms of enthalpy as , and since we obtain . Using the definition

we obtain for the specific heat at constant pressure

MP 3..9 Explanation of the above comparison between Diesel and Otto. Basically we can operate the diesel cycle at much higher compression ratio than the Otto cycle because only air is compressed and we don't run into the autoignition problem (knocking problem). Because of the higher compression ratios in the diesel engine we get higher efficiencies. MP 3..10 Would it be practical to run a Brayton cycle in reverse and use it as refrigerator? Yes. In fact people in Cryogenics use reversed Brayton cycles to cool down systems where very low temperatures are required (e.g., space applications, liquefaction of propellants). One major difference between a regular Brayton cycle (such as a jet-engine or a gas-turbine) and a reversed Brayton cycle is the working fluid. In order to make a reversed Brayton cycle practical we have to choose a working fluid that is appropriate for the application. Extremely low temperatures can be achieved when using a regenerator -- a heat exchanger that preheats the fluid before it enters the compressor and cools the fluid further down before it enters the turbine. In this configuration the fluid is expanded to much lower temperatures, and more heat can be absorbed from the cooling compartment.

MP 3..11 When flow is accelerated in a nozzle, doesn't that reduce the internal energy of the flow and therefore the enthalpy? Indeed both enthalpy and internal energy are reduced. The stagnation enthalpy is the quantity that is constant. MP 3..12 Why do we say that the combustion in a gas turbine engine is at constant pressure? This is an approximation, and a key question is indeed how accurate it is and what the justification is. The pressure change in the combustor can be analyzed using the one-dimensional compressible flow equations. The momentum equation is we obtain: , where c is the velocity. If we divide both sides by ,

where

is the speed of sound and

is the Mach number. ,

Changes in velocity are due to changes in density and in flow-through area as given by the one-dimensional continuity equation

Hence Differentiating,

Velocity changes are therefore related to area changes (geometry) and density changes (basically heat input). For a gas turbine combustion process the change in density is comparable with (a significant fraction of) the initial density and the area change is several times the initial area. This means that the change in velocity divided by the initial velocity is roughly of the order of magnitude of unity. The momentum equation thus tells us that for small Mach number (say 0.1) the ratio will be much less than one, so that the pressure can be approximated as constant. In reality the pressure does drop in the combustor, but the overall drop from inlet to exit is about 3-4%, small compared to the initial level of pressure, so that the approximation of constant pressure is a useful one.

The rapidity of the combustion process does not really have anything to do with this approximation. We could have a process, such as a nozzle, in which there was combustion at the same time that the pressure was dropping. As seen from the momentum equation, the heat addition does not ``directly'' affect the pressure -- changes in pressure are associated with changes in velocity. MP 3..13 Why is the Brayton cycle less efficient than the Carnot cycle? Consider the Brayton cycle and the corresponding work done as being approximated by a number of elementary Carnot cycles, as shown by the dashed lines in Figure 3.26. All of these Carnot cycles have the same pressure ratio, thus the same temperature ratio, and thus the same efficiency. The temperature ratio that figures into the efficiency of the elementary Carnot cycles is the inlet temperature divided by the compressor exit temperature, not the maximum cycle temperature, which is at the combustor exit. The basic reason for the lower efficiency is that heat is absorbed at an average temperature that is lower than the maximum temperature and rejected at an average temperature higher than the minimum temperature. We will come back to this important point (which has implications for all cycles), but if you cannot wait, see Chapter 6 of the notes.

Figure 3.26: Brayton cycle considered as a number of elementary Carnot cycles, all having the same pressure ratio and therefore the same temperature ratio, which is lower than the overall cycle temperature ratio, . MP 3..14 If the gas undergoes constant pressure cooling in the exhaust outside the engine, is that still within the system boundary? When we analyze the state changes as we trace them around the cycle, we are viewing the changes in a system, a mass of fixed identity. Thus we follow the mass as it moves through the device and the cooling of the gas outside the engine is happening to our system. MP 3..15 Does it matter what labels we put on the corners of the cycle or not?

It does not matter what labels we use on the corners of the cycle. A cycle is a series of processes. Independent of where you start in the cycle, it always brings you back to the state where you started. MP 3..16 Is the work done in the compressor always equal to the work done in the turbine plus work out (for a Brayton cyle)? NO. The work done in the compressor plus net work out equals the total turbine work. Using the 1st law, the net work we get out of the Brayton cycle is

(see notes for details). Rearranging the temperatures we can also write

Thus the net work is the difference between the enthalpy drop across the turbine (we get work from the turbine) and the enthalpy rise through the compressor (we have to supply work to the compressor). MP 3..17 What are the units of in ?

The units of power are J/s (kJ/s, MJ/s) or Watts (kW, MW). The mass flow is kg/s. The units of , work per unit mass, are thus J/kg. For the aeroengine, we can think of a given diameter (frontal area) as implying a given mass flow (think of a given Mach number and hence a given ratio of flow to choked flow). If so, for a given fan diameter power scales directly as work per unit mass. MP 3..18 Question about the assumptions made in the Brayton cycle for maximum efficiency and maximum work. We have first derived a general expression for the thermal efficiency of an ideal Brayton cycle (see Equation 3.10). The assumptions we made for the cycle were that both the compressor and turbine are ideal, such that they can be modeled adiabatic and reversible. We then looked at possible ideal Brayton cycles that would yield (A) maximum efficiency and (B) maximum work, keeping the assumptions of an ideal cycle (the assumptions of adiabatic and reversible compression and expansion stem from the choice of an ideal cycle). One way to construct an ideal Brayton cycle in the the inlet temperature and inlet pressure diagram is to choose

, the compressor pressure ratio

or temperature ratio , and the turbine inlet temperature . Apart from setting the inlet conditions (these mainly depend on the flight altitude and

Mach number and the day), we decided to fix the turbine inlet temperature (fixed by material technology or cost). So the only two ``floating'' cycle parameters that remain to be defined are the compressor exit temperature temperature and the turbine exit

. Looking at Equation 3.10 we know that the higher the

compressor temperature ratio the higher the thermal efficiency. So, for (A) maximum efficiency we would choose the compressor exit temperature as high as possible, that is in the limit . Constructing this cycle in the -

diagram and letting approach shows that the area enclosed by the cycle, or in other words the net work, becomes zero. Thus a cycle constructed for maximum efficiency, under the given inlet conditions and constraints on not very useful because we don't get any work out of it. For the derivation of notes for details. for maximum work (keeping , is

fixed as above), see

MP 3..19 You said that for a gas turbine engine modeled as a Brayton cycle the work done is , where is the heat added and is the heat rejected. Does this suggest that the work that you get out of the engine doesn't depend on how good your compressor and turbine are?...since the compression and expansion were modeled as adiabatic. Using the 1st law, the net work we get out of the cycle is

(see notes for details). Rearranging the temperatures we can also write

Thus the net work is the difference between the enthalpy drop across the turbine (we get work from the turbine) and the enthalpy rise through the compressor (we have to supply work to the compressor, this is done through the drive shaft that connects turbine and compressor). In class we analyzed an ideal Brayton cycle with the assumptions of adiabatic reversible compression and expansion processes, meaning that the work done by the turbine is the maximum work we can get from the given turbine (operating between and ), and the work needed to drive the given

compressor is the minimum work required. In the assumptions the emphasis is put on reversible rather than adiabatic. For real engines the assumption of adiabatic flow through the compressor and turbine still holds. This is an approximation -- the surface inside the compressor or turbine where heat can be transferred is much smaller than the mass flow of the fluid moving through the machine so that the heat transfer is negligible -- we will discuss the different concepts of heat transfer later in class. However the compression and expansion processes in real engines are irreversible due to non-ideal behavior and loss mechanisms occurring in the turbomachinery flow. Thus the thermal efficiency and work for a real jet engine with losses depend on the component efficiencies of turbine and compressor and are less than for an ideal jet engine. We will discuss these component efficiencies in more detail in class. MP 3..20 Why don't we like the numbers 1 and 2 for the stations? Why do we go 0-3? A common convention in the industry is that station 0 is far upstream, station 1 is after the shock in the inlet (if there is one), station 2 is at inlet to the compressor (after the inlet/diffuser) and station 3 is after the compressor. In class, when we examined the ramjet we considered no changes in stagnation pressure between 0 and 2, so I have used 0 as the initial state for the compression process. It would be more precise to differentiate between stations 0 and 2, and I will do this where appropriate. MP 3..21 For the Brayton cycle efficiency, why does The ramjet is operating as a Brayton cycle where . For the ramjet discussed in class the inlet temperature is and since there is no compressor (no moving parts) the only compression we get is from diffusion. We assumed isentropic diffusion in the diffuser and found for very low Mach numbers that the diffuser exit or combustor inlet temperature is . From the first law we know that for a steady, adiabatic flow where no work is done the stagnation enthalpy stays constant. Assuming a perfect gas with constant specific heats we thus get . So we can write for the ramjet thermal efficiency ?

4. Background to the Second Law of Thermodynamics


[VN Chapters 2, 3, 4] So far we have dealt largely with ideal situations involving quasi-static processes (i.e., frictionless pistons). We will now consider more general situations.

4.1 Reversibility and Irreversibility in Natural Processes


We wish to characterize the ``direction'' of natural processes; there is a basic ``directionality'' in nature. We start by examining a flywheel in a fluid filled insulated enclosure as shown in Figure 4.1.

Figure 4.1: Flywheel in insulated enclosure at initial and final states A question to be asked is whether we could start with state B and then let events proceed to state A? Why or why not? The first law does not prohibit this. The characteristics of state A are that the energy is in an organized form, the molecules in the flywheel have some circular motion, and we could extract some work by using the flywheel kinetic energy to lift a weight. In state B, in contrast, the energy is associated with disorganized motion on a molecular scale. The temperature of the fluid and flywheel are higher than in state A, so we could probably get some work out by using a Carnot cycle, but it would be much less than the work we could extract in state A. There is a qualitative difference between these states, which we need to be able to describe more precisely.

Muddy Points Why is the ability to do work decreased in B? How do we know? (MP 4.1)

Another example is a system composed of many bricks, half at a high temperature and half at a low temperature , as shown in Figure 4.2. With the bricks separated thermally, we have the ability to obtain work by running a cycle between the two temperatures. Suppose we put two bricks together. Using the first law we can write

where is the ``heat capacity'' . (For solids the heat capacities (specific heats) at constant pressure and constant volume are essentially the same.) We have lost the ability to get work out of these two bricks.

Figure 4.2: Bricks separated by a temperature difference Can we restore the system to the original state without contact with the outside? The answer is no. Can we restore the system to the original state with contact with the outside? The answer is yes. We could run a refrigerator to take heat out of one brick and put it into the other, but we would have to do work. We can think of the overall process involving the system (the two bricks in an insulated setting) and the surroundings (the rest of the universe) as:

System is changed, Surroundings are unchanged.

The composite system (system and the surroundings) is changed by putting the bricks together. The process is not reversible -- there is no way to undo the change and leave no mark on the surroundings. What is the measure of change in the surroundings? 1. Energy? This is conserved. 2. Ability to do work? This is decreased. The measurement and characterization of this type of change - of losing the ability to do work - is the subject of the second law of thermodynamics. [VW, S & B: 6.3-6.4]

4.2 Difference between Free Expansion of a Gas and Reversible Isothermal Expansion
The difference between reversible and irreversible processes is brought out through examination of the isothermal expansion of an ideal gas. The question to be asked is what is the difference between the ``free expansion'' of a gas and the isothermal expansion against a piston? To answer this, we address the steps that we would have to take to reverse, in other words, to undo the process. By free expansion, we mean the unrestrained expansion of a gas into a volume as shown in Figure 4.3. (The restrained expansion is shown in Figure 4.4.) Initially all the gas is in the volume designated as with the rest of the insulated

enclosure a vacuum. The total volume ( plus the evacuated volume) is . At a given time a hole is opened in the partition and the gas rushes through to fill the rest of the enclosure.

Figure 4.3: Free expansion

Figure 4.4: Expansion against a piston During the expansion there is no work exchanged with the surroundings because there is no motion of the boundaries4.1. The enclosure is insulated so there is no heat exchange. The first law tells us therefore that the internal energy is constant ( ). For an ideal gas, the internal energy is a function of temperature only so that the temperature of the gas before the free expansion and after the

expansion has been completed is the same. Characterizing the before and after states:

Before: State 1, After: State 2, ,

, .

, so there is no change in the surroundings. To restore the original state, i.e., to go back to the original volume at the same temperature ( at constant ) we can compress the gas isothermally (using work from an external agency). We can do this in a quasiequilibrium manner, with , as in Figure 4.5. If so the work that

we need to do is . We have evaluated the work in a reversible isothermal expansion (Eq. 3.1), and we can apply the arguments to the case of a reversible isothermal compression. The work done on the system to go from state ``2'' to state ``1'' is

Figure 4.5: Returning the free expansion to its initial condition From the first law, this amount of heat must also be rejected from the gas to the surroundings if the temperature of the gas is to remain constant. A schematic of the compression process, in terms of heat and work exchanged is shown in Figure 4.6.

Figure 4.6: Work and heat exchange in the reversible isothermal compression process At the end of the combined process (free expansion plus reversible compression): 1. The system has been returned to its initial state (no change in system state). 2. The surroundings (us!) did work on the system of magnitude 4.2 . 3. The surroundings received an amount of heat, , which is equal to . 4. The sum of all of these events is that we have converted an amount of work, , into an amount of heat, Joules. , with and numerically equal in

The net effect is the same as if we let a weight fall and pull a block along a rough surface, as in Figure 4.7. There is 100% conversion of work into heat.

Figure 4.7: 100% conversion of work into heat The results of the free expansion can be contrasted against a process of isothermal expansion against a pressure which is slightly different than that of the system, as shown in Figure 4.8.

Figure 4.8: Work and heat transfer in reversible isothermal expansion During the expansion, work is done on the surroundings of magnitude , where can be taken as the system pressure. As evaluated in Eq. (3.1), the magnitude of the work done by the system is . At the end of the isothermal expansion, therefore: 1. The surroundings have received work 2. The surroundings have given up heat, . , numerically equal to .

We now wish to restore the system to its initial state, just as we did in the free expansion. To do this we need to do work on the system and extract heat from the system, just as in the free expansion. In fact, because we are doing a transition between the same states along the same path, the work and heat exchange are the same as those for the compression process examined just above. The overall result when we have restored the system to the initial state, however, is quite different for the reversible expansion than for the free expansion. For the reversible expansion, the work we need to do on the system to compress it has the same magnitude as the work we received during the expansion process. Indeed, we could raise a weight during the expansion and then allow it to be lowered during the compression process. Similarly the heat put into the system by us (the surroundings) during the expansion process has the same magnitude as the heat received by us during the compression process. The result is that when the system has been restored back to its initial state, so have the surroundings. There is no trace of the overall process on either the system or the surroundings. That is another meaning of the word ``reversible.''

Muddy Points

With the isothermal reversible expansion is have ? (MP 4.2)

constant? If so, how can we

Why is the work done equal to zero in the free expansion? (MP 4.3) Is irreversibility defined by whether or not a mark is left on the outside environment? (MP 4.4)

4.3 Features of reversible processes


[VW, S & B: 6.3-6.4] Reversible processes are idealizations or models of real processes. One familiar and widely used example is Bernoulli's equation, which you saw in Unified. They are extremely useful for defining limits to system or device behavior, for enabling identification of areas in which inefficiencies occur, and in giving targets for design. An important feature of a reversible process is that, depending on the process, it represents the maximum work that can be extracted in going from one state to another, or the minimum work that is needed to create the state change. Let us consider processes that do work, so that we can show that the reversible one produces the maximum work of all possible processes between two states. For example, suppose we have a thermally insulated cylinder that holds an ideal gas, Figure 4.9. The gas is contained by a thermally insulated massless piston with a stack of many small weights on top of it. Initially the system is in mechanical and thermal equilibrium.

Figure 4.9: A piston with weights on top Consider the following three processes, shown in Figure 4.10: 1. All of the weights are removed from the piston instantaneously and the gas expands until its volume is increased by a factor of four (a free expansion).

2. Half of the weight is removed from the piston instantaneously, the system is allowed to double in volume, and then the remaining half of the weight is instantaneously removed from the piston and the gas is allowed to expand until its volume is again doubled. 3. Each small weight is removed from the piston one at a time, so that the pressure inside the cylinder is always in equilibrium with the weight on top of the piston. When the last weight is removed, the volume has increased by a factor of four.

Figure 4.10: Getting the most work out of a system requires that the work be extracted reversibly 4. Maximum work (proportional to the area under these curves) is obtained for the quasi-static expansion. *Note that there is a direct inverse relationship between the amount of work received from a process and the degree of irreversibility. To reiterate:

The work done by a system during a reversible process is the maximum work we can get. The work done on a system in a reversible process is the minimum work we need to do to achieve that state change.

A process must be quasi-static (quasi-equilibrium) to be reversible. This means that the following effects must be absent or negligible: 1. Friction: If we would have to do net work to bring the system from one volume to another and return it to the initial condition (recall Section 1.3.3.) 2. Free (unrestrained) expansion. 3. Heat transfer through a finite temperature difference .

Figure 4.11: Heat transfer across a finite temperature difference

Suppose we have heat transfer from a high temperature to a lower temperature as shown in Figure 4.11. How do we restore the situation to the initial conditions? One thought would be to run a Carnot refrigerator to get an amount of heat, , from the lower temperature reservoir to the higher temperature reservoir. We could do this but the surroundings, again us, would need to provide some amount of work (which we could find using our analysis of the Carnot refrigerator). The net (and only) result at the end of the combined process would be a conversion of an amount of work into heat. For reversible heat transfer from a heat reservoir to a system, the temperatures of the system and the reservoir must be . In other words the difference between the temperatures of the two entities involved in the heat transfer process can only differ by an infinitesimal amount, . While all natural processes are irreversible to some extent, it cannot be emphasized too strongly that there are a number of engineering situations where the effect of irreversibility can be neglected and the reversible process furnishes an excellent approximation to reality. The second law, which is the next topic we address, allows us to make a quantitative statement concerning the irreversibility of a given physical process.

Figure 4.12: Nicolas Sadi Carnot (1796-1832), an engineer and an officer in the French army. Carnot's work is all the more remarkable because it was made without benefit of the first law, which was not discovered until 30 years later. [Atkins, The Second Law]

Muddy Points Is heat transfer across a finite temperature difference only irreversible if no device is present between the two to harvest the potential difference? (MP 4.5)

4.4 Muddiest Points on Chapter 4


MP 4..1 Why is the ability to do work decreased in B? How do we know? In state A, the energy is in organized form and the molecules move along circular paths around the spinning flywheel. We could get work out this system by using all of the kinetic energy of the flywheel and for example lift a weight with it. The energy of the system in state B (flywheel not spinning) is associated with disorganized motion (on the molecular scale). The temperature in state B is higher than in state A. We could also extract work from state B by running for example an ideal Carnot cycle between and some heat reservoir at lower temperature. However the work we would get from this ideal Carnot cycle is less than the work we get from state A (all of the kinetic energy), because we must reject some heat when we convert heat into work (we cannot convert heat into 100% work). Although the energy of the system in state A is the same as in state B (we know this from 1st law) the ``organization'' of the energy is different, and thus the ability to do work is different. MP 4..2 With the isothermal reversible expansion, is so, how can we have ? constant? If

For a reversible process, if the external pressure were constant, there would need to be a force that pushed on the piston so the process could be considered quasi-equilibrium. This force could be us, it could be a system of weights, or it could be any other work receiver. Under these conditions the system pressure would not necessarily be near the external pressure but we would have . We can of course think of a situation in which the external pressure was varied so it was always close to the system pressure, but that is not necessary. MP 4..3 Why is the work done equal to zero in the free expansion? In this problem, the system is everything inside the rigid container. There is no change in volume, no `` ,'' so no work done on the surroundings. Pieces of the gas might be expanding, pushing on other parts of the gas, and doing work locally inside the container (and other pieces might be compressed and thus receive work) during the free expansion process, but we are considering the system as a whole, and there is no net work done. MP 4..4 Is irreversibility defined by whether or not a mark is left on the outside environment?

A process is irreversible when there is no way to undo the change without leaving a mark on the surroundings or ``the rest of the universe.'' In the example with the bricks, we could undo the change by putting a Carnot refrigerator between the bricks (both at after putting them together) and cooling one brick

down to and heating the other brick to to restore the initial state. To do this we have to supply work to the refrigerator and we will also reject some heat to the surroundings. Thus we leave a mark on the environment and the process is irreversible. MP 4..5 Is heat transfer across a finite temperature difference only irreversible if no device is present between the two to harvest the potential difference? If we have two heat reservoirs at different temperatures, the irreversibility associated with the transfer of heat from one to the other is indeed dependent on what is between them. If there is a copper bar between them, all the heat that comes out of the high temperature reservoir goes into the low temperature reservoir, with the result given in Section 5.5. If there were a Carnot cycle between them, some (not all) heat from the high temperature reservoir would be passed on to the low temperature reservoir, the process would be reversible, and work would be done. The extent to which the process is irreversible for any device can be assessed by computing the total entropy change (device plus surroundings) associated with the heat transfer.

5. The Second Law of Thermodynamics


[VN Chapter 5; VWB&S-6.3, 6.4, Chapter 7]

5.1 Concept and Statements of the Second Law (Why do we need a second law?)
The unrestrained expansion, or the temperature equilibration of the two bricks, are familiar processes. Suppose you are asked whether you have ever seen the reverse of these processes take place? Do two bricks at a medium temperature ever go to a state where one is hot and one is cold? Will the gas in the unrestrained expansion ever spontaneously return to occupying only the left side

of the volume? Experience hints that the answer is no. However, both these processes, unfamiliar though they may be, are compatible with the first law. In other words the first law does not prohibit their occurrence. There thus must be some other ``great principle'' that describes the direction of natural processes, that tells us which first law compatible processes will not be observed. This is contained in the second law. Like the first law, it is a generalization from an enormous amount of observation. There are several ways in which the second law of thermodynamics can be stated. Listed below are three that are often encountered. As described in class (and as derived in almost every thermodynamics textbook), although the three may not appear to have much connection with each other, they are equivalent. 1. No process is possible whose sole result is the absorption of heat from a reservoir and the conversion of this heat into work. [Kelvin-Planck statement of the second law]

Figure 5.1: This is not possible (Kelvin-Planck) 2. No process is possible whose sole result is the transfer of heat from a cooler to a hotter body. [Clausius statement of the second law]

Figure 5.2: For , this is not possible (Clausius) 3. There exists for every system in equilibrium a property called entropy, , which is a thermodynamic property of a system. For a reversible process, changes in this property are given by

The entropy change of any system and its surroundings, considered together, is positive and approaches zero for any process which approaches reversibility.

For an isolated system, i.e., a system that has no interaction with the surroundings, changes in the system have no effect on the surroundings. In this case, we need to consider the system only, and the first and second laws become:

For an isolated system the total energy ( ) is constant. The entropy can only increase or, in the limit of a reversible process, remain constant. The limit, or , represents the best that can be done. In thermodynamics, propulsion, and power generation systems we often compare performance to this limit to measure how close to ideal a given process is. All of these statements are equivalent, but 3 gives a direct, quantitative measure of the departure from reversibility. Entropy is not a familiar concept and it may be helpful to provide some additional rationale for its appearance. If we look at the first law,

the term on the left is a function of state, while the two terms on the right are not. For a simple compressible substance, however, we can write the work done in a reversible process as , so that

Two out of the three terms in this equation are expressed in terms of state variables. It seems plausible that we ought to be able to express the third term using state variables as well, but what are the appropriate variables? If so, the term should perhaps be viewed as analogous to where the parentheses denote an intensive state variable and the square brackets denote an extensive state variable. The second law tells us that the intensive variable is the temperature, , and the extensive state variable is the entropy, . The first law for a simple compressible substance in terms of state variables is thus (5..1)

Because Eq. 5.1 includes the second law, it is referred to as the combined first and second law. Because it is written in terms of state variables, it is true for all processes, not just reversible ones. We summarize below some attributes of entropy: 1. Entropy is a function of the state of the system and can be found if any two properties of the system are known, e.g. or or

. 2. is an extensive variable. The entropy per unit mass, or specific entropy, is . 3. The units of entropy are Joules per degree Kelvin (J/K). The units for specific entropy are J/K-kg. 4. For a system, , where the numerator is the heat given to the system and the denominator is the temperature of the system at the location where the heat is received. 5. for pure work transfer.

Muddy Points Why is always true? (MP 5.1)

What makes

different than

? (MP 5.2)

5.2 Axiomatic Statements of the Laws of Thermodynamics


5.2.1 Introduction
As a further aid in familiarization with the second law of thermodynamics and the idea of entropy, we draw an analogy with statements made previously concerning quantities that are closer to experience. In particular, we wish to present once more the Zeroth and First Laws of thermodynamics and use the same framework for the Second Law. In this so-called ``axiomatic formulation,'' the parallels between the Zeroth, First and Second Laws will be made explicit. 5.1

5.2.2 Zeroth Law


Section 1.3.2 presented this observation: Zeroth Law: There exists for every thermodynamic system in equilibrium a property called temperature. Equality of temperature is a necessary and sufficient condition for thermal equilibrium. The Zeroth law thus defines a property (temperature) and describes its behavior.

5.2.3 First Law


Observations also show that for any system there is a property called the energy. The First Law asserts that one must associate such a property with every system. First Law: There exists for every thermodynamic system a property called the energy. The change of energy of a system is equal to the mechanical work done on the system in an adiabatic process. In a non-adiabatic process, the change in energy is equal to the heat added to the system minus the mechanical work done by the system. On the basis of experimental results, therefore, one is led to assert the existence of two new properties, the temperature and internal energy, which do not arise in ordinary mechanics. In a similar way, a further remarkable relationship between heat and temperature will be established, and a new property, the entropy,

defined. Although this is a much less familiar property, it is to be stressed that the general approach is quite like that used to establish the Zeroth and First Laws. A general principle and a property associated with any system are extracted from experimental results. Viewed in this way, the entropy should appear no more mystical than the internal energy. The increase of entropy in a naturally occurring process is no less real than the conservation of energy.

5.2.4 Second Law


Although all natural processes must take place in accordance with the First Law, the principle of conservation of energy is, by itself, inadequate for an unambiguous description of the behavior of a system. Specifically, there is no mention of the familiar observation that every natural process has in some sense a preferred direction of action. For example, the flow of heat occurs naturally from hotter to colder bodies, in the absence of other influences, but the reverse flow certainly is not in violation of the First Law. So far as that law is concerned, the initial and final states are symmetrical in a very important respect. The Second Law is essentially different from the First Law; the two principles are independent and cannot in any sense be deduced from one another. Thus, the concept of energy is not sufficient, and a new property must appear. This property can be developed, and the Second Law introduced, in much the same way as the Zeroth and First Laws were presented. By examination of certain observational results, one attempts to extract from experience a law which is supposed to be general; it is elevated to the position of a fundamental axiom to be proved or disproved by subsequent experiments. Within the structure of classical thermodynamics, there is no proof more fundamental than observations. A statement which can be adopted as the Second Law of thermodynamics is: Second Law: There exists for every thermodynamic system in equilibrium an extensive scalar property called the entropy, , such that in an infinitesimal reversible change of state of the system, , where is the absolute temperature and

is the amount of heat received by the system. The entropy of a thermally insulated system cannot decrease and is constant if and only if all processes are reversible. As with the Zeroth and First Laws, the existence of a new property is asserted and its behavior is described.

5.2.5 Reversible Processes

In the course of this development, the idea of a completely reversible process is central, and we can recall the definition, ``a process is called completely reversible if, after the process has occurred, both the system and its surroundings can be wholly restored by any means to their respective initial states'' (first introduced in Section 1.3.3). Especially, it is to be noted that the definition does not, in this form, specify that the reverse path must be identical with the forward path. If the initial states can be restored by any means whatever, the process is by definition completely reversible. If the paths are identical, then one usually calls the process (of the system) reversible, or one may say that the state of the system follows a reversible path. In this path (between two equilibrium states 1 and 2), (i) the system passes through the path followed by the equilibrium states only, and (ii) the system will take the reversed path 2 to 1 by a simple reversal of the work done and heat added. Reversible processes are idealizations not actually encountered. However, they are clearly useful idealizations. For a process to be completely reversible, it is necessary that it be quasi-static and that there be no dissipative influences such as friction and diffusion. The precise (necessary and sufficient) condition to be satisfied if a process is to be reversible is the second part of the Second Law. The criterion as to whether a process is completely reversible must be based on the initial and final states. In the form presented above, the Second Law furnishes a relation between the properties defining the two states, and thereby shows whether a natural process connecting the states is possible.

Muddy Points What happens when all the energy in the universe is uniformly spread, i.e., entropy at a maximum? (MP 5.3)

5.3 Combined First and Second Law Expressions


The first law, written in a form that is always true:

For reversible processes only, work or heat may be rewritten as

Substitution leads to other forms of the first law true for reversible processes only:

(If the substance has other work modes, e.g., stress, strain, where is a pressure-like quantity, and and is a volume-like quantity.)

Substituting for both

in terms of state variables,

The above is always true because it is a relation between properties and is now independent of process. In terms of specific quantities:

The combined first and second law expressions are often more usefully written in terms of the enthalpy, or specific enthalpy, ,

Or, since

In terms of enthalpy (rather than specific enthalpy) the relation is

5.4 Entropy Changes in an Ideal Gas


[VW, S & B: 6.5- 6.6, 7.1] Many aerospace applications involve flow of gases (e.g., air) and we thus examine the entropy relations for ideal gas behavior. The starting point is form (a) of the combined first and second law,

For an ideal gas,

. Thus

Using the equation of state for an ideal gas ( ), we can write the entropy change as an expression with only exact differentials: (5..2)

We can think of Equation (5.2) as relating the fractional change in temperature to the fractional change of volume, with scale factors and ; if the volume increases without a proportionate decrease in temperature (as in the case of an adiabatic free expansion), then increases. Integrating Equation (5.2) between two states ``1'' and ``2'':

For a perfect gas with constant specific heats

In non-dimensional form (using

) (5..3)

Equation 5.3 is in terms of specific quantities. For

moles of gas,

This expression gives entropy change in terms of temperature and volume. We can develop an alternative form in terms of pressure and volume, which allows us to examine an assumption we have used. The ideal gas equation of state can be written as Taking differentials of both sides yields

Using the above equation in Eq. (5.2), and making use of the relations ; , we find

or

Integrating between two states 1 and 2 (5..4)

Using both sides of (5.4) as exponents we obtain

(5..5)

Equation (5.5) describes a general process. For the specific situation in which , i.e., the entropy is constant, we recover the expression . It was stated that this expression applied to a reversible, adiabatic process. We now see, through use of the second law, a deeper meaning to the expression, and to the concept of a reversible adiabatic process, in that both are characteristics of a constant entropy, or isentropic, process.

Muddy Points Why do you rewrite the entropy change in terms of ? (MP 5.4)

What is the difference between isentropic and adiabatic? (MP 5.5)

5.5 Calculation of Entropy Change in Some Basic Processes


1. Heat transfer from, or to, a heat reservoir. A heat reservoir (Figure 5.3) is a constant temperature heat source or sink. Because the temperature is uniform, there is no heat transfer across a finite temperature difference and the heat exchange is reversible. From the definition of entropy ( ),

where is the heat into the reservoir (defined here as positive if heat flows into the reservoir.)

Figure 5.3: Heat transfer from/to a heat reservoir 2. Heat transfer between two heat reservoirs The entropy change of the two reservoirs in Figure 5.4 is the sum of the entropy change of each. If the high temperature reservoir is at low temperature reservoir is at , the total entropy change is and the

Figure 5.4: Heat transfer between two reservoirs The second law says that the entropy change must be equal to or greater than zero. This corresponds to the statement that heat must flow from the higher temperature source to the lower temperature source. This is one of the statements of the second law given in Section 5.1.

Muddy Points In the single reservoir example, why can the entropy decrease? (MP 5.6) Why does the entropy of a heat reservoir change if the temperature stays the same? (MP 5.7) How can the heat transfer from or to a heat reservoir be reversible? (MP 5.8)

How can be less than zero in any process? Doesn't entropy always increase? (MP 5.9)

If for a reservoir, could you add still get the same ? (MP 5.10)

to any size reservoir and

3. Possibility of obtaining work from a single heat reservoir We can regard the process proposed in Figure 5.5 as the absorption of heat, , by a device or system, operating in a cycle, rejecting no heat, and producing work. The total entropy change is the sum of the change in the reservoir, the system or device, and the surroundings. The entropy change of the reservoir is . The entropy change of the device is zero, because we are considering a complete cycle (return to initial state) and entropy is a function of state. The surroundings receive work only so the entropy change of the surroundings is zero. The total entropy change is

Figure 5.5: Work from a single heat reservoir The total entropy change in the proposed process is thus less than zero,

which is not possible. The second law thus tells us that we cannot get work from a single reservoir only. The ``only'' is important; it means without any other changes occurring. This is the other statement of the second law we saw in Section 5.1.

Muddy Points What is the difference between the isothermal expansion of a piston and the (forbidden) production of work using a single reservoir? (MP 5.11) For the ``work from a single heat reservoir'' example, how do we know there is no ? (MP 5.12)

How does a cycle produce zero ? I thought that the whole thing about cycles was an entropy that the designers try to minimize. (MP 5.13) 4. Entropy changes in the ``hot brick problem''

[Temperature equalization of two bricks] used in reversible state transformations]

[Reservoirs

Figure 5.6: The ``Hot Brick'' Problem 5. We can examine in a more quantitative manner the changes that occurred when we put the two bricks together, as depicted in Figure 5.6(a). The process by which the two bricks come to the same temperature is not a reversible one, so we need to devise a reversible path. To do this imagine a large number of heat reservoirs at varying temperatures spanning the range , as in Figure 5.6(b). The bricks are put in contact with them sequentially to raise the temperature of one and lower the temperature of the other in a reversible manner. The heat exchange at any of these steps is entropy change is: . For the high temperature brick, the

6. 7. where is the heat capacity of the brick (J/kg). This quantity is less than zero. For the cold brick,

8. 9. The entropy change of the two bricks is

10. 11. The process is not reversible. 12. Difference between the free expansion and the reversible isothermal expansion of an ideal gas The essential difference between the free expansion in an insulated enclosure and the reversible isothermal expansion of an ideal gas can also be captured clearly in terms of entropy changes. For a state change from initial volume and temperature temperature , , to final volume and (the same)

the entropy change is

or, making use of the equation of state and the fact that isothermal process,

for an

This is the entropy change that occurs for the free expansion as well as for the isothermal reversible expansion processes - entropy changes are state changes and the two system final and end states are the same for both processes. For the free expansion:

There is no change in the entropy of the surroundings because there is no interaction between the system and the surroundings. The total entropy change is therefore,

There are several points to note from this result: 1. 2. so the process is not reversible.

; the equality between and is only for a reversible process. 3. There is a direct connection between the work needed to restore the system to the original state and the entropy change:

The quantity has a physical meaning as ``lost work'' in the sense of work which we lost the opportunity to utilize. We will make this connection stronger in Chapter 6. For the reversible isothermal expansion: The entropy is a state variable so the entropy change of the system is the same as before. In this case, however, heat is transferred to the system from the surroundings ( ) so that

The heat transferred from the surroundings, however, is equal to the heat received by the system: .

The total change in entropy (system plus surroundings) is therefore

The reversible process has zero total change in entropy.

Muddy Points On the example of free expansion versus isothermal expansion, how do we know that the pressure and volume ratios are the same? We know for each that and . (MP 5.14)

Where did

come from? (MP 5.15)

5.6 Muddiest Points on Chapter 5


MP 5..1 Why is always true? This is a relation between state variables. As such it is not path dependent, only depends on the initial and final states, and thus must hold no matter how we transition from initial state to final state. What is not always true, and what holds only for reversible processes, are the relations and . One

example of this is the free expansion where , but where the quantities and (and the integrals of these quantities) are not zero. MP 5..2 What makes The term different than ?

denotes the heat exchange during a reversible process. We use

the notation to denote heat exchange during any process, not necessarily reversible. The distinction between the two is important for the reason given above in Section 5.3. MP 5..3 What happens when all the energy in the universe is uniformly spread, i.e., entropy at a maximum? I quote from The Refrigerator and the Universe, by Goldstein and Goldstein:

The entropy of the universe is not yet at its maximum possible value and it seems to be increasing all the time. Looking forward to the future, Kelvin and Clausius foresaw a time when the maximum possible entropy would be reached and the universe would be at equilibrium forever afterward; at this point, a state called the ``heat death'' of the universe, nothing would happen forever after. The book also gives comments on the inevitability of this fate. MP 5..4 Why do you rewrite the entropy change in terms of ?

We have discussed the representation of thermodynamic changes in coordinates a number of times and it is familiar, as is the idea of the `` '' process. I want to relate this to the more general expression involving the entropy change Equation (5.5) to show (i) when the simple form applied and (ii) how valid an approximation it was. Using the entropy change, we now have a quantitative metric for doing just that. MP 5..5 What is the difference between isentropic and adiabatic? Isentropic means no change in entropy ( process with no heat transfer ( ). An adiabatic process is a

). We defined for reversible processes

. So generally an adiabatic process is not necessarily isentropic -only if the process is reversible and adiabatic we can call it isentropic. For example a real compressor can be assumed adiabatic but is operating with losses. Due to the losses the compression is irreversible. Thus the compression is not isentropic. MP 5..6 In the single reservoir example, why can the entropy decrease? When we looked at the single reservoir, our ``system'' was the reservoir itself. The example I did in class had heat leaving the reservoir, so that was negative. Thus the entropy change of the reservoir is also negative. The second law, however, guarantees that there is a positive change in entropy somewhere else in the surroundings that will be as large, or larger, than this decrease. MP 5..7 Why does the entropy of a heat reservoir change if the temperature stays the same? A heat reservoir is an idealization (like an ideal gas, a rigid body, an inviscid fluid, a discrete element mass-spring-damper system). The basic idea is that the heat capacity of the heat reservoir is large enough so that the transfer of heat in whatever problem we address does not appreciably alter the temperature of the

reservoir. In grappling with approximations such as this it is useful to think about extreme cases. Therefore, suppose the thermal reservoir is the atmosphere. The mass of the atmosphere is roughly kg (give or take an order of magnitude). Let us calculate the temperature rise due to the heat dumped into the atmosphere by a jet engine during a transcontinental flight. A large gas turbine engine might produce on the order of 100 MW of heat, so that the rise in atmospheric temperature, hour flight is given by , for the heat transfer associated with a 6

Substituting for the atmospheric mass and the specific heat gives a value for temperature change of roughly K. To a very good approximation, we can say that the temperature of this heat reservoir is constant and we can evaluate the entropy change of the reservoir as . MP 5..8 How can the heat transfer from or to a heat reservoir be reversible? We made the assumption that the heat reservoir is very large, and therefore it is a constant temperature heat source or sink. Since the temperature is uniform there is no heat transfer across a finite temperature difference and this heat exchange is reversible. We discussed this in the second example, ``Heat transfer between two heat reservoirs,'' in Section 5.5. MP 5..9 How can always increase? be less than zero in any process? Doesn't entropy

The second law says that the total entropy (system plus surroundings) always increases. (See Section 5.1). This means that either the system or the surroundings can have its entropy decrease if there is heat transfer between the two, although the sum of all entropy changes must be positive. For an isolated system, with no heat transfer to the surroundings, the entropy must always increase.

MP 5..10 If for a reservoir, could you add reservoir and still get the same ?

to any size

Yes, as long as the system you were adding heat to fulfilled the conditions for being a reservoir. MP 5..11 What is the difference between the isothermal expansion of a piston and the (forbidden) production of work using a single reservoir?

The difference is contained in the word sole in the Kelvin-Planck statement of the second law given in Section 5.1 of the notes. For the isothermal expansion the changes are: 1. The reservoir loses heat . ).

2. The system does work (equal in magnitude to 3. The system changes its volume and pressure.

4. The system changes its entropy (the entropy increases by For the ``forbidden'' process, 1. The reservoir loses heat .

).

2. The system does work ( ) and that's all the changes that there are. I leave it to you to calculate the total entropy changes (system plus surroundings) that occur in the two processes. MP 5..12 For the ``work from a single heat reservoir'' example, how do we know there is no ?

Our system was the heat reservoir itself. In the example we had heat leaving the reservoir, thus was negative and the entropy change of the reservoir was also negative. Using the second law, it is guaranteed that somewhere else in the surroundings a positive entropy change will occur that is as large or larger than the decrease of the entropy of the reservoir. MP 5..13 How does a cycle produce zero ? I thought that the whole thing about cycles was an entropy that the designers try to minimize. The change in entropy during a cycle is zero because we are considering a complete cycle (returning to initial state) and entropy is a function of state (holds for ideal and real cycles!). The entropy you are referring to is entropy that is generated in the components of a nonideal cycle. For example in a real jet engine we have a non-ideal compressor, a non-ideal combustor and also a non-ideal turbine. All these components operate with some loss and generate entropy -- this is the entropy that the designers try to minimize. Although the change in entropy during a nonideal cycle is zero, the total entropy change (cycle and heat reservoirs!) is

. Basically the entropy generated due to irreversibilities in the engine is additional heat rejected to the environment (to the lower heat reservoir). We will discuss this in detail in Section 6.1. MP 5..14 On the example of free expansion versus isothermal expansion, how do we know that the pressure and volume ratios are the same? We know for each that and .

During the free expansion no work is done and no heat is transferred (insulated system). Thus the internal energy stays constant and so does the temperature. This means that holds also for the free expansion and that the pressure and volume ratios are the same when comparing free expansion to reversible isothermal expansion.

MP 5..15 Where did

come from?

We were using the 1st and 2nd law combined (Gibbs) and in the example discussed there was no change in internal energy ( ). If we then integrate using moles of gas in volume and . (with being the number of being the universal gas constant) we obtain

6. Applications of the Second Law


[VN-Chapter 6; VWB&S-8.1, 8.2, 8.5, 8.6, 8.7, 8.8, 9.6]

6.1 Limitations on the Work that Can be Supplied by a Heat Engine


The second law enables us to make powerful and general statements concerning the maximum work that can be derived from any heat engine which operates in a cycle. To illustrate these ideas, we use a Carnot cycle which is shown schematically in Figure 6.1.

Figure 6.1: A Carnot cycle heat engine The engine operates between two heat reservoirs, exchanging heat high temperature reservoir at and changes of the two reservoirs are with the reservoir at with the

. The entropy

The same heat exchanges apply to the system, but with opposite signs: the heat received from the high temperature source is positive, and conversely. Denoting the heat transferred to the engines by subscript `` '', The total entropy change during any operation of the engine is

For a cyclic process, the third of these ( that ),

)is zero, and thus (remembering

(6..1)

For the engine we can write the first law as Or,

Hence, using (6.1)

The work of the engine can be expressed in terms of the heat received by the engine as

The upper limit of work that can be done occurs during a reversible cycle, for which the total entropy change ( ) is zero. In this situation:

Also, for a reversible cycle of the engine,

These constraints apply to all reversible heat engines operating between fixed temperatures. The thermal efficiency of the engine is

The Carnot efficiency is thus the maximum efficiency that can occur in an engine working between two given temperatures. We can approach this last point in another way. The engine work is given by

or,

The total entropy change can be written in terms of the Carnot cycle efficiency and the ratio of the work done to the heat absorbed by the engine. The latter is the efficiency of any cycle we can devise:

The second law says that the total entropy change is equal to or greater than zero. This means that the Carnot cycle efficiency is equal to or greater than the efficiency for any other cycle, with the total equality only occurring if .

Muddy Points So, do we lose the capability to do work when we have an irreversible process and entropy increases? (MP 6.1) Why do we study cycles starting with the Carnot cycle? Is it because it is easier to work with? (MP 6.2)

6.2 The Thermodynamic Temperature Scale


The considerations of Carnot cycles in this section have not mentioned the working medium. They are thus not limited to an ideal gas and hold for Carnot cycles with any medium. Earlier we derived the Carnot efficiency with an ideal gas as a medium and the temperature definition used in the ideal gas equation was not essential to the thermodynamic arguments. More specifically, we can define a thermodynamic temperature scale that is independent of the working medium. To see this, consider the situation shown below in Figure 6.2, which has three reversible cycles. There is a high temperature heat reservoir at and a

low temperature heat reservoir at . For any two temperatures , , the ratio of the magnitudes of the heat absorbed and rejected in a Carnot cycle has the same value for all systems.

Figure 6.2: Arrangement of heat engines to demonstrate the thermodynamic temperature scale We choose the cycles so and C. For a Carnot cycle is the same for A and C. Also is the same for B

Also

But

Hence

We thus conclude that

has the form

, and similarly

. The ratio of the heat exchanged is therefore

In general,

so that the ratio of the heat exchanged is a function of the temperature. We could choose any function that is monotonic, and one choice is the simplest: . This is the thermodynamic scale of temperature, . The temperature defined in this manner is the same as that for the ideal gas; the thermodynamic temperature scale and the ideal gas scale are equivalent.

6.3 Representation of Thermodynamic Processes in coordinates


It is often useful to plot the thermodynamic state transitions and the cycles in terms of temperature (or enthalpy) and entropy, , , rather than , . The maximum temperature is often the constraint on the process and the enthalpy changes show the work done or heat received directly, so that plotting in terms of these variables provides insight into the process. A Carnot cycle is shown below in these coordinates, in which it is a rectangle, with two horizontal, constant temperature legs. The other two legs are reversible and adiabatic, hence isentropic ( ), and therefore vertical in coordinates.

Figure 6.3: Carnot cycle in

coordinates

If the cycle is traversed clockwise, the heat added is

The heat rejected (from

to

) has magnitude

The work done by the cycle can be found using the first law for a reversible process:

We can integrate this last expression around the closed path traced out by the cycle:

However zero:

is an exact differential and its integral around a closed contour is

The work done by the cycle, which is represented by the term , the area enclosed by the closed contour in the represents the difference between the heat absorbed ( -

, is equal to plane. This area at the high

temperature) and the heat rejected ( work done through evaluation of

at the low temperature). Finding the is an alternative to computation of the

work in a reversible cycle from . Finally, although we have carried out the discussion in terms of the entropy, , all of the arguments carry over to the specific entropy, ; the work of the reversible cycle per unit mass is given by .

Muddy Points How does one interpret diagrams? (MP 6.3) and diagram? (MP 6.4) diagrams? (MP 6.5)

Is it always OK to ``switch''

What is the best way to become comfortable with What is a reversible adiabat physically? (MP 6.6)

6.4 Brayton Cycle in - Coordinates


The Brayton cycle has two reversible adiabatic (i.e., isentropic) legs and two reversible, constant pressure heat exchange legs. The former are vertical, but we need to define the shape of the latter. For an ideal gas, changes in specific enthalpy are related to changes in temperature by , so the shape of the cycle in an - plane is the same as in a - plane, with a scale factor of between the two. This suggests that a place to start is with the combined first and second law, which relates changes in enthalpy, entropy, and pressure:

On constant pressure curves and . The quantity desired is the derivative of temperature, , with respect to entropy, , at constant pressure:

and

. From the combined first and second law, and the relation between , this is (6..2)

The derivative is the slope of the constant pressure legs of the Brayton cycle on a - plane. For a given ideal gas (specific increases as . ) the slope is positive and

We can also plot the Brayton cycle in an - plane. This has advantages because changes in enthalpy directly show the work of the compressor and turbine and the heat added and rejected. The slope of the constant pressure legs in the plane is .

Note that the similarity in the shapes of the cycles in - and - planes is true for ideal gases only. As we will see when we examine two-phase cycles, the shapes look quite different in these two planes when the medium is not an ideal gas.

Figure 6.4: Ideal Brayton cycle as composed of many elementary Carnot cycles [Kerrebrock] Plotting the cycle in - coordinates also allows another way to address the evaluation of the Brayton cycle efficiency which gives insight into the relations between Carnot cycle efficiency and efficiency of other cycles. As shown in Figure 6.4, we can break up the Brayton cycle into many small Carnot cycles. The `` '' Carnot cycle has an efficiency of

where the indicated lower temperature is the heat rejection temperature for that elementary cycle and the higher temperature is the heat absorption temperature for that cycle. The upper and lower curves of the Brayton cycle, however, have constant pressure. All of the elementary Carnot cycles therefore have the same pressure ratio:

From the isentropic relations for an ideal gas, we know that pressure ratio, and temperature ratio, , are related by: .

The temperature ratios of any elementary cycle ``i'' are therefore the same and each of the elementary cycles has the same thermal efficiency. We only need to find the temperature ratio across any one of the cycles to find what the efficiency is. We know that the temperature ratio of the first elementary cycle is the ratio of compressor exit temperature to engine entry (atmospheric for an aircraft engine) temperature, in Figure 6.4. If the efficiency of all the elementary cycles has this value, the efficiency of the overall Brayton cycle (which is composed of the elementary cycles) must also have this value. Thus, as previously,

Figure 6.5: Arbitrary cycle operating between

A benefit of this view of efficiency is that it allows us a way to comment on the efficiency of any thermodynamic cycle. Consider the cycle shown in Figure 6.5, which operates between some maximum and minimum temperatures. We can break it up into small Carnot cycles and evaluate the efficiency of each. It can be seen that the efficiency of any of the small cycles drawn will be less than the

efficiency of a Carnot cycle between and . This graphical argument shows that the efficiency of any other thermodynamic cycle operating between these maximum and minimum temperatures has an efficiency less than that of a Carnot cycle.

Muddy Points If there is an ideal efficiency for all cycles, is there a maximum work or maximum power for all cycles? (MP 6.7)

6.4.1 Net work per unit mass flow in a Brayton cycle


In Section 3.7.1 we found the net work of a Brayton cycle in terms of heat transfer. Now that we have defined entropy, we can reexamine the net work using an enthalpy-entropy ( - ) diagram, Figure 6.6. The net mechanical work of the cycle is given by: where, by the first law,

If kinetic energy changes across the compressor and turbine are neglected, the temperature ratio, , across the compressor and turbine is related to the enthalpy changes:

The net work is thus

The turbine work is greater than the work needed to drive the compressor, as is evident on the ( - ) diagram.

Figure 6.6: Brayton cycle in enthalpy-entropy ( - ) representation showing compressor and turbine work

6.5 Irreversibility, Entropy Changes, and ``Lost Work''


Consider a system in contact with a heat reservoir during a reversible process. If there is heat absorbed by the reservoir at temperature , the change in

entropy of the reservoir is . In general, reversible processes are accompanied by heat exchanges that occur at different temperatures. To analyze these, we can visualize a sequence of heat reservoirs at different temperatures so that during any infinitesimal portion of the cycle there will not be any heat transferred over a finite temperature difference. During any infinitesimal portion, heat will be transferred between the . If is absorbed by the

system and one of the reservoirs which is at system, the entropy change of the system is

The entropy change of the reservoir is

The total entropy change of system plus surroundings is

This is also true if there is a quantity of heat rejected by the system. The conclusion is that for a reversible process, no change occurs in the total entropy produced, i.e., the entropy of the system plus the entropy of the surroundings: .

Figure 6.7: Irreversible and reversible state changes We now carry out the same type of analysis for an irreversible process, which takes the system between the same specified states as in the reversible process. This is shown schematically in Figure 6.7, with and denoting the irreversible and reversible processes. In the irreversible process, the system receives heat and does work process is . The change in internal energy for the irreversible

For the reversible process Because the state change is the same in the two processes (we specified that it was), the change in internal energy is the same. Equating the changes in internal energy in the above two expressions yields The subscript ``actual'' refers to the actual process (which is irreversible). The entropy change associated with the state change is (6..3)

If the process is not reversible, we obtain less work (see IAW notes) than in a reversible process, , so that for the irreversible process,

There is no equality between the entropy change

and the quantity

for an irreversible process. The equality is only applicable for a reversible process. The change in entropy for any process that leads to a transformation between an initial state ``a'' and a final state ``b'' is therefore

where is the heat exchanged in the actual process. The equality only applies to a reversible process. The difference represents work we could have obtained, but . In terms of this

did not. It is referred to as lost work and denoted by quantity we can write,

(6..4)

The content of Equation (6.4) is that the entropy of a system can be altered in two ways: (i) through heat exchange and (ii) through irreversibilities. The lost work ( in Equation (6.4)) is always greater than zero, so the only way to decrease the entropy of a system is through heat transfer. To apply the second law we consider the total entropy change (system plus surroundings). If the surroundings are a reservoir at temperature , with which the system exchanges heat,

The total entropy change is

The quantity (

) is the entropy generated due to irreversibility.

Yet another way to state the distinction we are making is (6..5)

The lost work is also called dissipation and noted . Using this notation, the infinitesimal entropy change of the system becomes:

or

Equation (6.5) can also be written as a rate equation, (6..6)

Either of Equation (6.5) or (6.6) can be interpreted to mean that the entropy of the system, , is affected by two factors: the flow of heat and the appearance

of additional entropy, denoted by , due to irreversibility6.1. This additional entropy is zero when the process is reversible and always positive when the process is irreversible. Thus, one can say that the system develops sources which create entropy during an irreversible process. The second law asserts that sinks of entropy are impossible in nature, which is a more graphic way of saying that and are positive definite (always greater than zero), or zero in the special case of reversible processes.

The term

which is associated with heat transfer to the system, can be interpreted as a flux of entropy. The boundary is crossed by heat and the ratio of this heat flux to temperature can be defined as a flux of entropy. There are no restrictions on the sign of this quantity, and we can say that this flux either contributes towards, or drains away, the system's entropy. During a reversible process, only this flux can affect the entropy of the system. This terminology suggests that we interpret entropy as a kind of weightless fluid, whose quantity is conserved (like that of matter) during a reversible process. During an irreversible process, however, this fluid is not conserved; it cannot disappear, but rather is created by sources throughout the system. While this interpretation should not be taken too literally, it provides an easy mode of expression and is in the same category of concepts such as those associated with the phrases ``flux of energy'' or ``sources of heat.'' In fluid mechanics, for example, this graphic language is very effective and there should be no objections to copying it in thermodynamics.

Muddy Points Do we ever see an absolute variable for entropy? So far, we have worked with deltas only (MP 6.8)

I am confused as to

as opposed to

.(MP 6.9)

For irreversible processes, how can we calculate (MP 6.10)

if not equal to

6.6 Entropy and Unavailable Energy (Lost Work by Another Name)


Consider a system consisting of a heat reservoir at atmosphere) at in surroundings (the . . The surroundings are equivalent to a second reservoir at

For an amount of heat,

, transferred from the reservoir, the maximum work we

could derive is times the thermal efficiency of a Carnot cycle operated between these two temperatures: (6..7)

Only part of the heat transferred can be turned into work, in other words only part of the heat energy is available to be used as work. Suppose we transferred the same amount of heat from the reservoir directly to another reservoir at a temperature the quantity of heat, . The maximum work available from is

, before the transfer to the reservoir at

The maximum amount of work available after the transfer to the reservoir at

is

There is an amount of energy that could have been converted to work prior to the irreversible heat transfer process of magnitude ,

or

However,

is the entropy gain of the reservoir at

and (

) is the

entropy decrease of the reservoir at . The amount of energy, , that could have been converted to work (but now cannot be) can therefore be written in terms of entropy changes and the temperature of the surroundings as

The situation just described is a special case of an important principle concerning entropy changes, irreversibility and the loss of capability to do work. We thus now develop it in a more general fashion, considering an arbitrary system undergoing an irreversible state change, which transfers heat to the surroundings (for example the atmosphere), which can be assumed to be at constant temperature, . The change in internal energy of the system during the state change is . The change in entropy of the surroundings is (with transfer to the system) the heat

Now consider restoring the system to the initial state by a reversible process. To do this we need to do work, quantity of heat, , on the system and extract from the system a

. (We did this, for example, in ``undoing'' the free

expansion process.) The change in internal energy is (with the quantities and both regarded, in this example, as positive for work done by the surroundings and heat given to the surroundings) 6.2. In this reversible process, the entropy of the surroundings is changed by

For the combined changes (the irreversible state change and the reversible state change back to the initial state), the energy change is zero because the energy is a function of state, Thus,

For the system, the overall entropy change for the combined process is zero, because the entropy is a function of state,

The total entropy change is thus only reflected in the entropy change of the surroundings: The surroundings can be considered a constant temperature heat reservoir and their entropy change is given by

We also know that the total entropy change, for system plus surroundings is,

The total entropy change is associated only with the irreversible process and is related to the work in the two processes by

The quantity represents the extra work required to restore the system to the original state. If the process were reversible, we would not have needed any extra work to do this. It represents a quantity of work that is now unavailable because of the irreversibility. The quantity can also be interpreted as the work that the system would have done if the original process were reversible. From either of these perspectives we can identify as the quantity we denoted previously as , representing lost work. The lost work in any irreversible process can therefore be related to the total entropy change (system plus surroundings) and the temperature of the surroundings by

To summarize the results of the above arguments for processes where heat can be exchanged with the surroundings at 1. :

represents the difference between work we actually obtained and work that would be done during a reversible state change. It is the extra work that would be needed to restore the system to its initial state. ; ; . .

2. For a reversible process, 3. For an irreversible process, 4.

is the energy that becomes unavailable for work during an irreversible process.

Muddy Points Is path dependent? (MP 6.11) the and going from the final state back to the initial

Are and state? (MP 6.12)

6.7 Examples of Lost Work in Engineering Processes


1. Lost work in Adiabatic Throttling: Entropy and Stagnation Pressure Changes

Figure 6.8: Adiabatic Throttling 2. A process we have encountered before is adiabatic throttling of a gas, by a valve or other device as shown in Figure 6.8. The velocity is denoted by . There is no shaft work and no heat transfer and the flow is steady. Under these conditions we can use the first law for a control volume (the

Steady Flow Energy Equation) to make a statement about the conditions upstream and downstream of the valve:

3. 4. where is the stagnation enthalpy, corresponding to a (possibly fictitious) state with zero velocity. The stagnation enthalpy is the same at stations 1 and 2 if , even if the flow processes are not reversible. .

5. For a perfect gas with constant specific heats, and The relation between the static and stagnation temperatures is:

6.

7. where is the speed of sound and is the Mach number, . In deriving this result, use has only been made of the first law, the equation of state, the speed of sound, and the definition of the Mach number. Nothing has yet been specified about whether the process of stagnating the fluid is reversible or irreversible. 8. When we define the stagnation pressure, however, we do it with respect to isentropic deceleration to the zero velocity state. For an isentropic process

9. 10. The relation between static and stagnation pressures is

11.

Figure 6.9: Static and stagnation pressures and temperatures 12. The stagnation state is defined by in , . In addition,

. The static and stagnation states are shown coordinates in Figure 6.9.

13. Stagnation pressure is a key variable in propulsion and power systems. To see why, we examine the relation between stagnation pressure, stagnation temperature, and entropy. The form of the combined first and second law that uses enthalpy is (6..8) 14.

Figure 6.10: Stagnation and static states 15. This holds for small changes between any thermodynamic states and we can apply it to a situation in which we consider differences between stagnation states, say one state having properties and the other

having properties (see Figure 6.10). The corresponding static states are also indicated. Because the entropy is the same at static and stagnation conditions, needs no subscript. Writing (6.8) in terms of stagnation conditions yields

16. 17. Both sides of the above are perfect differentials and can be integrated as

18. 19. For a process with , the stagnation enthalpy, and hence the stagnation temperature, is constant. In this situation, the stagnation pressure is related directly to the entropy as, (6..9) 20.

Figure 6.11: Losses reflected in changes in stagnation pressure when 21. Figure 6.11 shows this relation on a - diagram. We have seen that the entropy is related to the loss, or irreversibility. The stagnation pressure plays the role of an indicator of loss if the stagnation temperature is constant. The utility is that it is the stagnation pressure (and temperature) which are directly measured, not the entropy. The throttling process is a representation of flow through inlets, nozzles, stationary turbomachinery blades, and the use of stagnation pressure as a measure of loss is a practice that has widespread application. 22. Equation (6.9) can be put in several useful approximate forms. First, we note that for aerospace applications we are (hopefully!) concerned with low loss devices, so that the stagnation pressure change is small compared to the inlet level of stagnation pressure,

23. 24. Expanding the logarithm (using ),

25. 26. or

27. 28. Another useful form is obtained by dividing both sides by and taking the limiting forms of the expression for stagnation pressure in the limit of low Mach number ( ). Doing this, we find:

29. 30. The quantity on the right can be interpreted as the change in the ``Bernoulli constant'' for incompressible (low Mach number) flow. The quantity on the left is a non-dimensional entropy change parameter, with the term now representing the loss of mechanical energy associated with the change in stagnation pressure. 31. To summarize: 1. For many applications the stagnation temperature is constant and the change in stagnation pressure is a direct measure of the entropy increase. 2. Stagnation pressure is the quantity that is actually measured so that linking it to entropy (which is not measured) is useful. 3. We can regard the throttling process as a ``free expansion'' at constant temperature from the initial stagnation pressure to the final stagnation pressure. We thus know that, for the process, the work we need to do to bring the gas back to the initial state is , which is the ``lost work'' per unit mass.

Muddy Points Why do we find stagnation enthalpy if the velocity never equals zero in the flow? (MP 6.13)

Why does

remain constant for throttling? (MP 6.14)

32. Adiabatic Efficiency of a Propulsion System Component (Turbine)

Figure 6.12: Schematic of turbine and associated thermodynamic representation in - coordinates 33. A schematic of a turbine and the accompanying thermodynamic diagram are given in Figure 6.12. There is a pressure and temperature drop through the turbine and it produces work. There is no heat transfer so the expressions that describe the overall shaft work and the shaft work per unit mass are (6..10) (6..11) 34. 35. If the difference in the kinetic energy at inlet and outlet can be neglected, Equation (6.11) reduces to 36. 37. The adiabatic efficiency of the turbine is defined as

38. 39. The performance of the turbine can be represented in an - plane (similar to a - plane for a perfect gas with constant specific heats) as shown in Figure 6.12. From the figure the adiabatic efficiency is

40. 41. The adiabatic efficiency can therefore be written as

42.

43. The non-dimensional term represents the departure from isentropic (reversible) processes and hence a loss. The quantity is the enthalpy difference for two states along a constant pressure line (see diagram). From the combined first and second laws, for a constant pressure process, small changes in enthalpy are related to the entropy change by , or approximately, 44. 45. The adiabatic efficiency can thus be approximated as

46. 47. The quantity represents a useful figure of merit for fluid machinery inefficiency due to irreversibility. 48.

Muddy Points 49. How do you tell the difference between shaft work/power and flow work in a turbine, both conceptually and mathematically? (MP 6.15) 50. Isothermal Expansion with Friction

Figure 6.13: Isothermal expansion with friction 51. In a more general look at the isothermal expansion, we now drop the restriction to frictionless processes. As seen in Figure 6.13, work is done to overcome friction. If the kinetic energy of the piston is negligible, a balance of forces tells us that 52. 53. During the expansion, the piston and the walls of the container will heat up because of the friction. The heat will be (eventually) transferred to the atmosphere; all frictional work ends up as heat transferred to the surrounding atmosphere. 54.

55. The amount of heat transferred to the atmosphere due to the frictional work only is thus,

56. 57. The entropy change of the atmosphere (considered as a heat reservoir)

due to the frictional work is 58. The engine operates in a cycle and the entropy change for the complete cycle is zero (because entropy is a state variable). Therefore,

59. 60. The total entropy change is

61. 62. Suppose we had an ideal reversible engine working between these same two temperatures, which extracted the same amount of heat, , from the

high temperature reservoir, and rejected heat of magnitude to the low temperature reservoir. The work done by this reversible engine is 63. 64. For the reversible engine the total entropy change over a cycle is

65. 66. Combining the expressions for work and for the entropy changes, 67.

68. The entropy change for the irreversible cycle can therefore be written as

69. 70. The difference in work that the two cycles produce is proportional to the entropy that is generated during the cycle: 71. 72. The second law states that the total entropy generated is greater than zero for an irreversible process, so that the reversible work is greater than the actual work of the irreversible cycle. 73. An ``engine effectiveness,'' , can be defined as the ratio of the actual work obtained divided by the work that would have been delivered by a reversible engine operating between the two temperatures and :

74. 75. The departure from a reversible process is directly reflected in the entropy change and the decrease in engine effectiveness. 76.

Muddy Points 77. Why does ? (MP 6.17)

78. In discussing the terms ``closed system'' and ``isolated system,'' can you assume that you are discussing a cycle or not? (MP 6.18) 79. Does a cycle process have to have ? (MP 6.19)

80. In a real heat engine, with friction and losses, why is ? (MP 6.20) 81. Propulsive Power and Entropy Flux

still 0 if

The final example in this section combines a number of ideas presented in this subject and in Unified in the development of a relation between entropy generation and power needed to propel a vehicle. Figure 6.15 shows an aerodynamic shape (airfoil) moving through the atmosphere at a constant velocity. A coordinate system fixed to the vehicle has been adopted so that we see the airfoil as fixed and the air far away from the airfoil moving at a velocity . Streamlines of the flow have been sketched, as has the velocity distribution at station ``0'' far upstream and station ``d'' far downstream. The airfoil has a wake, which mixes with the surrounding air and grows in the downstream direction. The extent of the wake is also indicated. Because of the lower velocity in the wake the area between the stream surfaces is larger downstream than upstream.

Figure 6.15: Airfoil with wake and control volume for analysis of propulsive power requirement We use a control volume description and take the control surface to be defined by the two stream surfaces and two planes at station 0 and station d. This is useful in simplifying the analysis because there is no flow across the stream surfaces. The area of the downstream plane control surface is broken into , which is the area outside the wake and , which is the area occupied by wake fluid, i.e., fluid that has suffered viscous losses. The control surface is also taken far enough away from the vehicle so that the static pressure can be considered uniform. For fluid which is not in the wake (no viscous forces), the momentum equation is

Uniform static pressure therefore implies uniform velocity, so that on the velocity is equal to the upstream value, . The downstream velocity profile is actually continuous, as indicated. It is approximated in the analysis as a step change to make the algebra a bit simpler. (The conclusions apply to the more general velocity profile as well and we would just need to use integrals over the wake instead of the algebraic expressions below.) The equation expressing mass conservation for the control volume is (6..12)

The vertical face of the control surface is far downstream of the body. By this station, the wake fluid has had much time to mix and the velocity in the wake is close to the free stream value, . We can thus write, (6..13)

(We chose our control surface so the condition

was upheld.)

The integral momentum equation (control volume form of the momentum equation) can be used to find the drag on the vehicle: (6..14)

There is no pressure contribution in Eq. (6.14) because the static pressure on the control surface is uniform. Using the form given for the wake velocity and expanding the terms in the momentum equation we obtain

(6..15)

The last term in the right hand side of the momentum equation, , is small by virtue of the choice of control surface and we can neglect it. Doing this and grouping the terms on the right hand side of Eq. (6.15) in a different manner, we have

The terms in the square brackets on both sides of this equation are the continuity equation multiplied by curly bracketed terms as . They thus sum to zero leaving the

(6..16)

The wake mass flow is . All this flow has a velocity defect (compared to the free stream) of , so that the defect in flux of momentum (the mass flow in the wake times the velocity defect) is, to first order in ,

The combined first and second law gives us a means of relating the entropy and velocity:

The pressure is uniform ( ) at the downstream station. There is no net shaft work or heat transfer to the wake so that the mass flux of stagnation enthalpy is constant. We can also approximate that the condition of constant stagnation enthalpy holds locally on all streamlines. Applying both of these to the combined first and second law yields

For the present situation,

, so that (6..17)

In Equation (6.17) the upstream temperature is used because differences between wake quantities and upstream quantities are small at the downstream control station. The entropy can be related to the drag as (6..18)

The quantity is the entropy flux (mass flux times the entropy increase per unit mass; in the general case we would express this by an integral over the locally varying wake velocity and density). The power needed to propel the vehicle is the product of ,

. From Eq. (6.18), this can be related to the entropy flux in the wake to yield a compact expression for the propulsive power needed in terms of the wake entropy flux: (6..19)

This amount of work is dissipated per unit time in connection with sustaining the vehicle motion. Equation (6.19) is another demonstration of the relation between lost work and entropy generation, in this case manifested as power that needs to be supplied because of dissipation in the wake.

Muddy Points Is it safe to say that entropy is the tendency for a system to go into disorder? (MP 6.21)

6.8 Some Overall Comments on Entropy, Reversible and Irreversible Processes


[Mainly excerpted (with some alterations) from: Engineering Thermodynamics, William C. Reynolds and Henry C. Perkins, McGraw-Hill Book Company, 1977.]

6.8.1 Entropy
1. Entropy is a thermodynamic property that measures the degree of randomization or disorder at the microscopic level. The natural state of affairs is for entropy to be produced by all processes. 2. A macroscopic feature which is associated with entropy production is a loss of ability to do useful work. Energy is degraded to a less useful form, and it is sometimes said that there is a decrease in the availability of energy. 3. Entropy is an extensive thermodynamic property. In other words, the entropy of a complex system is the sum of the entropies of its parts. 4. The notion that entropy can be produced, but never destroyed, is the second law of thermodynamics.

6.8.2 Reversible and Irreversible Processes


Processes can be classed as reversible or irreversible. The concept of a reversible process is an important one which directly relates to our ability to recognize, evaluate, and reduce irreversibilities in practical engineering processes. Consider an isolated system. The second law says that any process that would reduce the entropy of the isolated system is impossible. Suppose a process takes place within the isolated system in what we shall call the forward direction. If the change in state of the system is such that the entropy increases for the forward process, then for the backward process (that is, for the reverse change in

state) the entropy would decrease. The backward process is therefore impossible, and we therefore say that the forward process is irreversible. If a process occurs, however, in which the entropy is unchanged by the forward process, then it would also be unchanged by the reverse process. Such a process could go in either direction without contradicting the second law. Processes of this latter type are called reversible. The key idea of a reversible process is that it does not produce any entropy. Entropy is produced in irreversible processes. All real processes (with the possible exception of superconducting current flows) are in some measure irreversible, though many processes can be analyzed quite adequately by assuming that they are reversible. Some processes that are clearly irreversible include: mixing of two gases, spontaneous combustion, friction, and the transfer of energy as heat from a body at high temperature to a body at low temperature. Recognition of the irreversibilities in a real process is especially important in engineering. Irreversibility, or departure from the ideal condition of reversibility, reflects an increase in the amount of disorganized energy at the expense of organized energy. The organized energy (such as that of a raised weight) is easily put to practical use; disorganized energy (such as the random motions of the molecules in a gas) requires ``straightening out'' before it can be used effectively. Further, since we are always somewhat uncertain about the microscopic state, this straightening can never be perfect. Consequently the engineer is constantly striving to reduce irreversibilities in systems, in order to obtain better performance.

6.8.3 Examples of Reversible and Irreversible Processes


Processes that are usually idealized as reversible include:

Frictionless movement Restrained compression or expansion Energy transfer as heat due to infinitesimal temperature nonuniformity Electric current flow through a zero resistance Restrained chemical reaction Mixing of two samples of the same substance at the same state.

Processes that are irreversible include: Movement with friction Unrestrained expansion Energy transfer as heat due to large temperature non uniformities

Electric current flow through a non zero resistance Spontaneous chemical reaction Mixing of matter of different composition or state.

6.9 Effect of Departures from Ideal Behavior -- Real Cycle behavior


To conclude this chapter, we will now improve our estimates of cycle performance by including the effects of irreversibility. We will use the Brayton cycle as an example. What are the sources of non-ideal performance and departures from reversibility?

Losses (entropy production) in the compressor and the turbine. Stagnation pressure decrease in the combustor. Heat transfer.

We take into account here only irreversibility in the compressor and in the turbine. Because of these irreversibilities, we need more work, (the changes in kinetic energy from inlet to exit of the compressor are neglected), to drive the compressor than in the ideal situation. We also get less work, , back from the turbine. The consequence, as can be inferred from Figure 6.16, is that the net work from the engine is less than in the cycle with ideal components.

Figure 6.16: Gas turbine engine (Brayton) cycle showing effect of departure from ideal behavior in compressor and turbine To develop a quantitative description of the effect of these departures from reversible behavior, consider a perfect gas with constant specific heats and neglect kinetic energy at the inlet and exit of the turbine and compressor. We define the turbine adiabatic efficiency as

where is specified to be at the same pressure ratio as . There is a similar metric for the compressor, the compressor adiabatic efficiency:

again for the same pressure ratio. Note that for the turbine the ratio is the actual work delivered divided by the ideal work, whereas for the compressor the ratio is the ideal work needed divided by the actual work required. These are not thermal efficiencies, but rather measures of the degree to which the compression and expansion approach the ideal processes. We now wish to find the net work done in the cycle and the efficiency. The net work is given either by the difference between the heat received and rejected or the work of the compressor and turbine, where the convention is that heat received is positive and heat rejected is negative and work done is positive and work absorbed is negative.

The thermal efficiency is

We need to calculate From the definition of

, ,

With

Similarly, by the definition

we can find

The thermal efficiency can now be found:

With

and

or

There are several non-dimensional parameters that appear in this expression for thermal efficiency. We list these in the two sections below and show their effects in accompanying figures.

6.9.1 Parameters reflecting design choices

6.9.2 Parameters reflecting the ability to design and execute efficient components

In addition to efficiency, net rate of work is a quantity we need to examine,

Putting this in a non-dimensional form,

[Non-dimensional work as a function of cycle pressure ratio for different values of turbine entry temperature divided by compressor entry temperature]

[Overall cycle efficiency as a function of pressure ratio for different values of turbine entry temperature divided by

compressor entry temperature] [Overall cycle efficiency as a function of cycle pressure ratio for different component efficiencies]

Figure 6.17: Non-dimensional power and efficiency for a non-ideal gas turbine engine [from Cumpsty, Jet Propulsion] Trends in net power and efficiency are shown in Figure 6.17 for parameters typical of advanced civil engines. Some points to note in the figure:

For any

the optimum pressure ratio

for maximum

is not the highest that can be achieved, as it is for the ideal Brayton cycle. The ideal analysis is too idealized in this regard. The highest efficiency also occurs closer to the pressure ratio for maximum power than in the case of an ideal cycle. Choosing this as a design criterion will therefore not lead to the efficiency penalty inferred from ideal cycle analysis. There is a strong sensitivity to the component efficiencies. For example, for ideal value. , the cycle efficiency is roughly two-thirds of the

The maximum power occurs at a value of than that for max

or pressure ratio

less

(this trend is captured by ideal analysis). are strongly dependent on

The maximum power and maximum the maximum temperature, .

A scale diagram of a Brayton cycle with non-ideal compressor and turbine behaviors, in terms of temperature-entropy ( - ) and pressure-volume ( coordinates is given below as Figure 6.18.

[] [] Figure 6.18: Scale diagram of non-ideal gas turbine cycle. Nomenclature is shown in the figure. Pressure ratio and turbine efficiencies , , , compressor [from Cumpsty, Jet Propulsion]

Muddy Points Isn't it possible for the mixing of two gases to go from the final state to the initial state? If you have two gases in a box, they should eventually separate by density, right? (MP 6.22)

How can

be the maximum turbine inlet temperature? (MP 6.23)

When there are losses in the turbine that shift the expansion in - diagram to the right, does this mean there is more work than ideal since the area is greater? (MP 6.24) For an afterburning engine, why must the nozzle throat area increase if the temperature of the fluid is increased? (MP 6.25) Why doesn't the pressure in the afterburner go up if heat is added? (MP 6.26) Why is the flow in the nozzle choked? (MP 6.27) What's the point of having a throat if it creates a retarding force? (MP 6.28) Why isn't the stagnation temperature conserved in this steady flow? (MP 6.29)

6.10 Muddiest Points on Chapter 6


MP 6..1 So, do we lose the capability to do work when we have an irreversible process and entropy increases? Absolutely. We will see this in a more general fashion very soon. The idea of lost work is one way to view what ``entropy is all about.'' MP 6..2 Why do we study cycles starting with the Carnot cycle? Is it because it is easier to work with? Carnot cycles are the best we can do in terms of efficiency. We use the Carnot cycle as a standard against which all other cycles are compared. We will see in class that we can break down a general cycle into many small Carnot cycles. Doing this we can gain insight in which direction the design of efficient cycles should go. MP 6..3 How does one interpret h-s diagrams?

I find - diagrams useful, especially in dealing with propulsion systems, because the difference in stagnation can be related (from the Steady Flow Energy Equation) to shaft work and heat input. For processes that just have shaft work (compressors or turbines) the change in stagnation enthalpy is the shaft work. For processes that just have heat addition or rejection at constant pressure, the change in stagnation enthalpy is the heat addition or rejection. MP 6..4 Is it always OK to ``switch'' T-s and h-s diagrams? No! This is only permissible for perfect gases with constant specific heats. We will see, when we examine cycles with liquid-vapor mixtures, that the diagrams and the - diagrams look different. MP 6..5 What is the best way to become comfortable with T-s diagrams? I think working with these diagrams may be the most useful way to achieve this objective. In doing this, the utility of using these coordinates (or - coordinates) should also become clearer. I find that I am more comfortable with - or diagrams than with - diagrams, especially the latter because it conveys several aspects of interest to propulsion engineers: work produced or absorbed, heat produced or absorbed, and loss. MP 6..6 What is a reversible adiabat physically? Let's pick an example process involving a chamber filled with a compressible gas and a piston. We assume that the chamber is insulated (so no heat transfer to or from the chamber) and the process is thus adiabatic. Let us also assume that the piston is ideal, such that there is no friction between the walls of the chamber and the piston. The gas is at some temperature . We now push the piston in and compress the gas. The internal energy of the system will then increase by the amount of work we put in and the gas will heat up and be at higher pressure. If we now let the piston expand again, it will return to its original position (no friction, ideal piston) and the work we took from the environment will be returned (we get the exact same amount of work back and leave no mark on the environment). Also, the temperature and the pressure of the gas return to the initial values. We thus have an adiabatic reversible process. For both compression and expansion we have no change in entropy of the system because there is no heat transfer and also no irreversibility. If we now draw this process in the - or - diagram we get a vertical line since the entropy stays constant: or and we can also call this process an isentropic process. MP 6..7 If there is an ideal efficiency for all cycles, is there a maximum work or maximum power for all cycles?

Yes. As with the Brayton cycle example, we could find the maximum as a function of the appropriate design parameters. MP 6..8 Do we ever see an absolute variable for entropy? So far, we have worked with deltas only. It is probably too strong a statement to say that for ``us'' the changes in entropy are what matters, but this has been my experience for the type of problems aerospace engineers work on. Some values of absolute entropy are given in Table A.8 in SB&VW. We will also see, in the lectures on Rankine cycles, that the entropy of liquid water at a temperature of and a pressure of has been specified as zero for problems involving two-phase (steam and water) behavior.

MP 6..9 I am confused as to

as opposed to

Both of these are true and both can always be used. The first is the definition of entropy. The second is a statement of how the entropy behaves. Section 6.5 attempts to make the relationship clearer through the development of the equality . if not equal

MP 6..10 For irreversible processes, how can we calculate to ?

We need to define a reversible process between the two states in order to calculate the entropy (see MP 6.9). See VN Chapter 5 (especially) for discussion of entropy or Section 6.5. If you are still in difficulty, come and see me. MP 6..11 Is path dependent?

No. Entropy is a function of state (see Gibbs) and thus is path independent. For example we could have three different paths connecting the same two states and therefore have the same change in entropy

MP 6..12 Are the initial state?

and

the

and

going from the final state back to

Yes. We have an irreversible process from state 1 to state 2. We then used a reversible process to restore the initial state 1 (we had to do work on the system and extract heat from the system).

MP 6..13 Why do we find stagnation enthalpy if the velocity never equals zero in the flow? The stagnation enthalpy (or stagnation temperature) is a useful reference quantity. Unlike the static temperature it does not vary along a streamline in an adiabatic flow, even if irreversible. It was thus the natural reference temperature to use in describing the throttling process. In addition, changes in stagnation enthalpy (or stagnation temperature) are direct representations of the shaft work or heat associated with a fluid component. The static enthalpy is not, unless we assume that changes in are small. Measurement of stagnation temperature thus allows direct assessment of shaft work in a turbine or compressor, for example. MP 6..14 Why does remain constant for throttling?

Because for a steady adiabatic flow with no shaft work done the Steady Flow Energy Equation yields constant stagnation enthalpy even though the flow processes might not be reversible (see notes). For a perfect gas with constant specific heats, , thus the stagnation temperature remains constant.

MP 6..15 How do you tell the difference between shaft work/power and flow work in a turbine, both conceptually and mathematically? Let us look at the expansion of a flow through a turbine using both the control mass approach and the control volume approach. Using the control mass approach we can model the situation by tracking of air as follows: state 1 --

before the expansion we have of air upstream of the turbine. We then push the gas into the turbine and expand it through the blade rows. After the expansion we take of air out of the turbine and the mass of air is downstream of the turbine -- state 2. The work done by the gas is work done by the turbine (blades moved around by the gas) plus the work done by pressures (flow work). Using the first law we can then write for the change of internal energy of of air:

When entering the turbine, the fluid has to push the surroundings out of the way to make room for itself (it has volume and is at ) -- the work to do this is

. When leaving the turbine the fluid is giving up room and the work to keep that volume the shaft work at pressure is freed; thus . We can then write for

The right hand side of the above equation is the change in enthalpy . This is another example to show how useful enthalpy is (enthalpy is the total energy of a fluid: the internal energy plus the extra energy that it is credited with by having a volume at pressure ). The shaft work output by the turbine is equal to the change in enthalpy (enthalpy contains the flow work!). We can also solve this problem by using the 1st law in general form (control volume approach):

Note that in this form the flow work is buried in already! For this turbine, we can drop the unsteady term on the left and neglect heat fluxes (adiabatic turbine), shear work and piston work (no pistons but blades, so we keep the shaft work). Further we assume that changes in potential energy and kinetic energy are negligible and we obtain for air

We obtain the same result as before: . MP 6..16 Is the entropy change in the equations two lines above the total entropy change? If so, why does it say ?

The entropy change in question is the entropy change due to the heat produced by friction only.

MP 6..17 Why does

When we wrote this equality, we were considering a system that was returned to its original state, so that there were no changes in any of the system properties. All evidence of irreversibility thus resides in the surroundings. MP 6..18 In discussing the terms ``closed system'' and ``isolated system,'' can you assume that you are discussing a cycle or not? The terms closed system and isolated system have no connection to whether we are discussing a cycle or not. They are attributes of a system (any system), whether undergoing cyclic behavior, one-way transitions, or just sitting there. MP 6..19 Does a cycle process have to have ?

Entropy is a state function. If the process is cyclic, initial and final states are the same. So, for a cyclic process, . MP 6..20 In a real heat engine, with friction and losses, why is if ? still

The change in entropy during a real cycle is zero because we are considering a complete cycle (returning to initial state) and entropy is a function of state (holds for both ideal and real cycles!). Thus if we integrate around

the real cycle we will obtain . What essentially happens is that all irreversibilities ( 's) are turned into additional heat that is rejected to the environment. The amount of heat rejected in the real ideal cycle be larger than the amount of heat rejected in an ideal cycle : is going to

We will see this better using the surroundings (heat reservoirs) is

diagram. The change of entropy of the .

So

even for real cycles, but

. MP 6..21 Is it safe to say that entropy is the tendency for a system to go into disorder? Entropy can be given this interpretation from a statistical perspective, and this provides a different and insightful view of this property. At the level in which we have engaged the concept, however, we focus on the macroscopic properties of systems, and there is no need to address the idea of order and disorder; as we will see, entropy is connected to the loss of our ability to do work, and that is sufficient to make it a concept of great utility for the evaluation and design of engineering systems. We will look at this in a later lecture. If you are interested in pursuing this, places to start might be Great Ideas in Physics by Lightman (paperback book by an MIT professor), or Modern Thermodynamics by Kondepudi and Prigogine. MP 6..22 Isn't it possible for the mixing of two gases to go from the final state to the initial state? If you have two gases in a box, they should eventually separate by density, right? Let us assume that gas is oxygen and gas is nitrogen. When the membrane breaks the entire volume will be filled with a mixture of oxygen and nitrogen. This may be considered as a special case of an unrestrained expansion, for each gas undergoes an unrestrained expansion as it fills the entire volume. It is impossible for these two uniformly mixed gases to separate without help from the surroundings or environment. A certain amount of work is necessary to separate the gases and to bring them back into the left and right chambers.

MP 6..23 How can

be the maximum turbine inlet temperature?

I agree that the is a temperature ratio. If we assume constant ambient temperature then this ratio reflects the maximum cycle temperature. The main point was to emphasize that the higher your turbine inlet temperature the higher your power and efficiency levels. MP 6..24 When there are losses in the turbine that shift the expansion in - diagram to the right, does this mean there is more work than ideal since the area is greater? We have to be careful when looking at the area enclosed by a cycle or underneath a path in the - diagram. Only for a reversible cycle, the area enclosed is the work done by the cycle (see notes Section 6.3). Looking at the

Brayton cycle with losses in compressor and turbine the net work is the difference between the heat absorbed and the heat rejected (from the 1 st law). The heat absorbed can be found by integrating along the heat addition process. The heat rejected during the cycle with losses in compressor and turbine is larger than in the ideal cycle (look at the area underneath the path where heat is rejected, this area is larger than when there are no losses (when ). See also muddy point 6.1. So we get less net work if irreversibilities are present. It is sometimes easier to look at work and heat (especially shaft work for turbines and compressors) in the - diagram because the enthalpy difference between two states directly reflects the shaft work (remember, enthalpy includes the flow work!) and / or heat transfer. MP 6..25 For an afterburning engine, why must the nozzle throat area increase if the temperature of the fluid is increased? The Mach number of the flow is unity at the throat with and without the afterburner lit. The ratio of static pressure to stagnation pressure at the throat is thus the same with and without the afterburner lit. The ratio of static temperature to stagnation temperature at the throat is thus the same with and without the afterburner lit.

The flow through the throat is

The flow through the throat thus scales as

From what we have said, however, the pressure at the throat is the same in both cases. Also, we wish to have the mass flow the same in both cases in order to have the engine operate at near design conditions. Putting these all together,

plus use of the idea that the ratio of stagnation to static temperature at the throat is the same for both cases gives the relation

The necessary area to pass the flow is proportional to the square root of the stagnation temperature. If too much fuel is put into the afterburner, the increase in area cannot be met and the flow will decrease. This can stall the engine, a serious consequence for a single engine fighter. MP 6..26 Why doesn't the pressure in the afterburner go up if heat is added? From discussions after lecture, the main point here seems to be that the process of heat addition in the afterburner, or the combustor, is not the same as heat addition to a gas in a box. In that case the density (mass/volume) would be constant and, from , increasing the temperature would increase the pressure. In a combustor, the geometry is such that the pressure is approximately constant; this happens because the fluid has the freedom to expand so the density decreases. From the equation temperature goes up, the density must go down. MP 6..27 Why is the flow in the nozzle choked? As seen in Unified, choking occurs when the stagnation to static pressure ratio gets to a certain value, for gas with of 1.4. Almost all jet aircraft aircraft operate at flight conditions such that this is achieved. If you are not comfortable with the way in which the concepts of choking are laid out in the Unified notes, please see me and I can give some references. MP 6..28 What's the point of having a throat if it creates a retarding force? As shown in Unified, to accelerate the flow from subsonic to supersonic, i.e., to create the high velocities associated with high thrust, one must have a converging-diverging nozzle, and hence a throat. MP 6..29 Why isn't the stagnation temperature conserved in this steady flow? Heat is added in the afterburner, so the stagnation temperature increases. if the

7. Interpretation of Entropy on the Microscopic Scale -- The Connection between Randomness and Entropy 7.1 Entropy Change in Mixing of Two Ideal Gases
Consider an insulated rigid container of gas separated into two halves by a heat conducting partition so the temperature of the gas in each part is the same. One side contains air, the other side another gas, say argon, both regarded as ideal gases. The mass of gas in each side is such that the pressure is also the same. The entropy of this system is the sum of the entropies of the two parts: . Suppose the partition is taken away so the gases are free to diffuse throughout the volume. For an ideal gas, the energy is not a function of volume, and, for each gas, there is no change in temperature. (The energy of the overall system is unchanged, the two gases were at the same temperature initially, so the final temperature is the same as the initial temperature.) The entropy change of each gas is thus the same as that for a reversible isothermal expansion from the initial specific volume specific volume, . For a mass to the final

of ideal gas, the entropy change is

. The entropy change of the system is (7..1)

Equation (7.1) states that there is an entropy increase due to the increased volume that each gas is able to access. Examining the mixing process on a molecular level gives additional insight. Suppose we were able to see the gas molecules in different colors, say the air molecules as white and the argon molecules as red. After we took the partition away, we would see white molecules start to move into the red region and,

similarly, red molecules start to come into the white volume. As we watched, as the gases mixed, there would be more and more of the different color molecules in the regions that were initially all white and all red. If we moved further away so we could no longer pick out individual molecules, we would see the growth of pink regions spreading into the initially red and white areas. In the final state, we would expect a uniform pink gas to exist throughout the volume. There might be occasional small regions which were slightly more red or slightly more white, but these fluctuations would only last for a time on the order of several molecular collisions. In terms of the overall spatial distribution of the molecules, we would say this final state was more random, more mixed, than the initial state in which the red and white molecules were confined to specific regions. Another way to say this is in terms of ``disorder;'' there is more disorder in the final state than in the initial state. One view of entropy is thus that increases in entropy are connected with increases in randomness or disorder. This link can be made rigorous and is extremely useful in describing systems on a microscopic basis. While we do not have scope to examine this topic in depth, the purpose of this chapter is to make plausible the link between disorder and entropy through a statistical definition of entropy.

7.2 Microscopic and Macroscopic Descriptions of a System


The microscopic description of a system is the complete description of each particle in this system. In the above example, the microscopic description of the gas would be the list of the state of each molecule: position and velocity in this problem. It would require a great deal of data for this description; there are roughly molecules in a cube of air one centimeter on a side at room temperature and pressure. The macroscopic description, which is in terms of a few (two!) properties is thus far more accessible and useable for engineering applications, although it is restricted to equilibrium states. To address the description of entropy on a microscopic level, we need to state some results concerning microscopic systems. These results and the computations and arguments below are taken almost entirely from the excellent discussion in Chapter 6 of Engineering Thermodynamics by Reynolds and Perkins7.1. For a given macroscopic system, there are many microscopic states. A key idea from quantum mechanics is that the states of atoms, molecules, and entire systems are discretely quantized. This means that a system of particles under certain constraints, like being in a box of a specified size, or having a fixed total

energy, can exist in a finite number of allowed microscopic states. This number can be very big, but it is finite. The microstates of the system keep changing with time from one quantum state to another as molecules move and collide with one another. The probability for the system to be in a particular quantum state is defined by its quantum-state probability . The set of the is called the distribution of probability. The sum of the probabilities of all the allowed quantum states must be unity, hence for any time , (7..2)

When the system reaches equilibrium, the individual molecules still change from one quantum state to another. In equilibrium, however, the system state does not change with time; so the probabilities for the different quantum states are independent of time. This distribution is then called the equilibrium distribution, and the probability can be viewed as the fraction of time a system spends in the quantum state. In what follows, we limit consideration to equilibrium states. We can get back to macroscopic quantities from the microscopic description using the probability distribution. For instance, the macroscopic energy of the system would be the weighted average of the successive energies of the system (the energies of the quantum states); the energies are weighted by the relative time the system spends in the corresponding microstates. In terms of probabilities, the average energy, , is (7..3)

where

is the energy of a quantum state.

The probability distribution provides information on the randomness of the equilibrium quantum states. For example, suppose the system can only exist in three states (1, 2 and 3). If the distribution probability is

the system is in quantum state 1 and there is no randomness. If we were asked what quantum state the system is in, we would be able to say it is always in state 1. If the distribution were

or

the randomness would not be zero and would be equal in both cases. We would be more uncertain about the instantaneous quantum state than in the first situation. Maximum randomness corresponds to the case where the three states are equally probable:

In this case, we can only guess the instantaneous state with

probability.

7.3 A Statistical Definition of Entropy


The list of the is a precise description of the randomness in the system, but the number of quantum states in almost any industrial system is so high this list is not useable. We thus look for a single quantity, which is a function of the , that gives an appropriate measure of the randomness of a system. As shown below, the entropy provides this measure. There are several attributes that the desired function should have. The first is that the average of the function over all of the microstates should have an extensive behavior. In other words the microscopic description of the entropy of a system , composed of parts and should be given by

(7..4)

Second is that entropy should increase with randomness and should be largest for a given energy when all the quantum states are equiprobable. The average of the function over all the microstates is defined by (7..5)

where the function is to be found. Suppose that system has microstates and system has microstates. The entropies of systems and , are defined by

In Equations (7.5) and (7.6), the term system is in state and system expressions in Equations (7.6),

means the probability of a microstate in which . For Equation (7.4) to hold given the

is in state

(7..7)

The function and

must be such that this is true regardless of the values of the probabilities because . in the first part of

. This will occur if

To verify this, make this substitution in the expression for Equation (7.6c) (assume the probabilities , and split the log term): and

are independent, such that

(7..8)

Rearranging the sums, (7.8) becomes (7..9)

Because (7..10)

the square brackets in the right hand side of Equation (7.9) can be set equal to unity, with the result written as (7..11)

This reveals the top line of Equation (7.7) to be the same as the bottom line, for any , , , provided that

is a logarithmic function. Reynolds and Perkins show that

the most general

is

, where

is an arbitrary constant. Because the

are less than unity, the constant is chosen to be negative to make the entropy positive. Based on the above, a statistical definition of entropy can be given as: (7..12)

The constant

is known as the Boltzmann constant, (7..13)

The value of

is (another wonderful result!) given by (7..14)

where

is the universal gas constant,

and

is

Avogadro's number, molecules per mol. Sometimes is called the gas constant per molecule. With this value for , the statistical definition of entropy is identical with the macroscopic definition of entropy.

7.4 Connection between the Statistical Definition of Entropy and Randomness


We need now to examine the behavior of the statistical definition of entropy as regards randomness. Because a uniform probability distribution reflects the largest randomness, a system with allowed states will have the greatest

entropy when each state is equally likely. In this situation, the probabilities become (7..15)

where

is the total number of microstates. The entropy is thus

(7..16)

Equation (7.16) states that the larger the number of possible states the larger the entropy. The behavior of the entropy stated in Equation (7.16) can be summarized as follows: 1. is maximum when is maximum, which means many permitted quantum states, hence much randomness, 2. is minimum when is minimum. In particular, for , there is no randomness and . These trends are in accord with our qualitative ideas concerning randomness. Equation (7.16) is carved on Boltzmann's tombstone (he died in 1906 in Vienna). We can also examine the additive property of entropy with respect to probabilities. If we have two systems, and , which are viewed as a combined system, , the quantum states for the combined system are the combinations of the quantum states from and . The quantum state where is in its state and is in its state would have a probability because the two probabilities are independent. The number of probabilities for the combined system, system is , is thus defined by . The entropy of the combined

(7..17)

Equation (7.16) is sometimes taken as the basic definition of entropy, but it should be remembered that it is only appropriate when each quantum state is

equally likely. Equation (7.12) is more general and applies equally for equilibrium and non-equilibrium situations. A simple numerical example shows trends in entropy changes and randomness for a system which can exist in three states. Consider the five probability distributions

The first distribution has no randomness. For the second, we know that state 3 is never found. Distributions (iii) and (iv) have progressively greater uncertainty about the distribution of states and thus higher randomness. Distribution (v) has the greatest randomness and uncertainty and also the largest entropy.

7.5 Numerical Example of the Approach to the Equilibrium Distribution


Reynolds and Perkins give a numerical example which illustrates the above concepts and also the tendency of a closed isolated system to tend to equilibrium. The starting point is a system in an initial microscopic state that is not an equilibrium distribution. We expect the system will change quantum state, with disorder, randomness growing until they reach the equilibrium values. The specific system to be studied is composed of 10 particles , , , ..., , each of which can exist in one of 5 states, of energies 0, 1, 2, 3, 4. The system is isolated and has a total energy of 30. The total energy remains unchanged during the evolution of the microscopic states. Some of the allowed states are shown in Figure 7.1

Figure 7.1: Some allowed states of the system in the numerical example. Note each state has a total energy of 30. [Reynolds and Perkins, 1977]

Figure 7.2: Constant energy state groups [Reynolds and Perkins, 1977] For ten particles, 4 energy states, and a total energy of 30, there are 72,403 possible quantum states (4 states are indicated in Figure 7.1). However, there are only 23 possible distributions in terms of the number of particles having a given energy as shown in Figure 7.2. For example, states 2 and 3 in Figure 7.1 are two different quantum states, but they represent the same group (22) in Figure 7.2. The allowed state groups If the quantum-state probabilities are equal, each quantum state has a probability of 1/72,403. The probabilities of each group are thus directly proportional to the number of quantum states in this group. For instance, group 22 has 90 quantum states, so its probability is . We now know what the equilibrium distribution of probabilities is. We now address the time evolution of a system to the equilibrium state. To see this, we start a system from one of the 22 non-equilibrium groups and track the behavior over time. A way to examine the process is to consider what happens if two particles interact, doing this numerically for the instantaneous quantum state. The two particles are free to change energy as long as the total energy of the system is conserved. This may or may not end up by changing the state group (the particles could interact and

only switch states). There are 45 possible pairs for this interaction (there are possible ways to carry out the interaction, but two of them, say interactions between and and and , are the same), and we assume that any of them is equally likely to happen. If the system is initially in state 1 of Figure 7.1, it is in group 23 of Figure 7.2. For each of the 45 pairs, there are two interactions that take the system to group 22, and one that leaves the system unchanged. (For interactions between and , say, the result can be that and have their energy unchanged, that loses energy and gains energy, or that gains energy and loses energy. In the first of these, the system will remain in group 23. In the second and third it will move to group 22.) Hence the transition probability from group 23 to group 22 is , and the transition probability from 23 to 23 is .

Figure 7.3: Transition probabilities (probability for transition from initial group to final group) in numerical experiment with isolated system [Reynolds and Perkins, 1977] For the other groups, the transitions are more complicated, but can be found numerically, with the results shown in Figure 7.3. The numerical experiments were carried out with the system initially in state 23 and with successive interactions chosen randomly in accordance with the transition probabilities of

Figure 7.3. The experiment was repeated 10,000 times, with a different group history traced out each time and, again, the system energy maintained at 30. The fraction of the experiments in which each group occurred at time was used to calculate the group probabilities for the distribution at that time. at each time. The entropy was then found

Figure 7.4: Evolution of the probability distribution with time (interaction number) [Reynolds and Perkins, 1977]

Figure 7.4 shows the evolution of some of the with time (the unit of time is the interaction number for the calculations) starting from group 23. After roughly ten interactions, the probabilities have reached a steady-state level, which are the equilibrium probabilities from Figure 7.2.

Figure 7.5: Entropy for the system as a function of time [Reynolds and Perkins, 1977] The computed entropy is given in Figure 7.5 as a function of time. It increases to the equilibrium value with the same sort of behavior as the probability distribution. The interactions allow the system to change groups. The transition probabilities are large for groups with high equilibrium probabilities. There is one additional aspect of the behavior that is brought out in the text. This is the difference in overall probabilities between the order of transitions. The probability of a transition sequence is the product of the individual step transition probabilities. The transition 23-22-12-9-1 thus has the probability: . The reverse transition, 1-912-22-23 has the probability: . There is an enormous probability that the system will move towards (and persist in) quantum state groups that have high equilibrium probabilities. Once a system has moved out of group 23, there is little likelihood that it will ever return. Further, for engineering systems, which have not 10 particles, but upwards of , the difference between transitions and their reverses are much more marked, and the probability is overwhelming that the distribution will be a quantum state with a broad distribution of particle energies.

7.6 Summary and Conclusions


1. Entropy as defined from a microscopic point of view is a measure of randomness in a system. 2. The entropy is related to the probabilities states of the system by of the individual quantum

where

, the Boltzmann constant, is given by

3. For a system in which there are

quantum states, all of which are equally ), the entropy is given by

probable (for which the probability is

The more quantum states, the more the randomness and uncertainty that a system is in a particular quantum state. 4. From the statistical point of view there is a finite, but exceedingly small possibility that a system that is well mixed could suddenly ``unmix'' and that all the air molecules in the room could suddenly come to the front half of the room. The unlikelihood of this is well described by Denbigh [Principles of Chemical Equilibrium, 1981] in a discussion of the behavior of an isolated system: ``In the case of systems containing an appreciable number of atoms, it becomes increasingly improbable that we shall ever observe the system in a non-uniform condition. For example, it is calculated that the probability of a relative change of density, , of only in of air is

smaller than and would not be observed in trillions of years. Thus, according to the statistical interpretation the discovery of an appreciable and spontaneous decrease in the entropy of an isolated system, if it is separated into two parts, is not impossible, but exceedingly improbable. We repeat, however, that it is an absolute impossibility to know when it will take place.''

5. The definition of entropy in the form arises in other aerospace fields, notably that of information theory. In this context, the constant is taken as unity and the entropy becomes a dimensionless measure of the uncertainty represented by a particular message. There is no underlying physical connection with thermodynamic entropy, but the underlying uncertainty concepts are the same. 6. The presentation of entropy in this subject is focused on the connection to macroscopic variables and behavior. These involve the definition of entropy given in Chapter 5 of the notes and the physical link with lost work, neither of which makes any mention of molecular (microscopic) behavior. The approach in other sections of the notes is only connected to these macroscopic processes and does not rely at all upon the microscopic viewpoint. Exposure to the statistical definition of entropy,

however, is helpful as another way not only to answer the question of ``What is entropy?'' but also to see the depth of this fundamental concept and the connection with other areas of technology.

8. Power Cycles with Two-Phase Media (Vapor Power Cycles)


[SB&VW - Chapter 3, Chapter 11, Sections 11.1 to 11.7] In this chapter, we examine cycles that use two-phase media as the working fluid. These can be combined with gas turbine cycles to provide combined cycles which have higher efficiency than either alone. They can also be used by themselves to provide power sources for both terrestrial and space applications. The topics to be covered are 1. Behavior of two-phase systems (equilibrium, pressure temperature relations), 2. Carnot cycles with two-phase media, 3. Rankine cycles, and 4. Combined cycles.

8.1 Behavior of Two-Phase Systems


The definition of a phase, as given by SB&VW, is ``a quantity of matter that is homogeneous throughout.'' Common examples of systems that contain more than one phase are a liquid and its vapor and a glass of ice water. A system which has three phases is a container with ice, water, and water vapor. We wish to find the relations between phases and the relations that describe the change of phase (from solid to liquid, or from liquid to vapor) of a pure substance, including the work done and the heat transfer. To start we consider a system consisting of a liquid and its vapor in equilibrium, which are enclosed in a container under a moveable piston, as shown in Figure 8.1. The system is maintained at constant temperature through contact with a heat reservoir at temperature , so there can be heat transfer to or from the system.

[] [] [] Figure 8.1: Two-phase system in contact with constant temperature heat reservoir

Figure 8.2:

relation for a liquid-vapor system

For a pure substance, as shown in Figure 8.2, there is a one-to-one correspondence between the temperature at which vaporization occurs and the pressure. These values are called the saturation pressure and saturation temperature (see Ch. 3 in SB&VW). This means there is an additional constraint for a liquid-vapor mixture, in addition to the equation of state. The consequence is that we only need to specify one variable to determine the state of the system. For example, if we specify then is set. In summary, for two phases in equilibrium, . If both phases are present, any quasi-static process at constant is also at constant . Let us examine the pressure-volume behavior of a liquid-vapor system at constant temperature. For a single-phase ideal gas we know that the curve would be . For the two-phase system the curve looks quite different, as indicated in Figure 8.3.

Figure 8.3:

diagram for two-phase system showing isotherms

Several features of the figure should be noted. First, there is a region in which liquid and vapor can coexist, bounded by the liquid saturation curve on the left and the vapor saturation curve on the right. This is roughly dome-shaped and is thus often referred to as the ``vapor dome.'' Outside of this regime, the equilibrium state will be a single phase. The regions of the diagram in which the system will be in the liquid and vapor phases respectively are indicated. Second is the steepness of the isotherms in the liquid phase, due to the small compressibility of most liquids. Third, the behavior of isotherms at temperatures below the ``critical point'' (see below) in the region to the right of the vapor dome approach those of an ideal gas as the pressure decreases, and the ideal gas relation is a good approximation in this region. The behavior shown is found for all the isotherms that go through the vapor dome. At a high enough temperature, specifically at a temperature corresponding to the pressure at the peak of the vapor dome, there is no transition from liquid to vapor and the fluid goes continuously from a liquid-like behavior to a gas-type behavior. This behavior is unfamiliar, mainly because the temperatures and pressures are not ones that we typically experience; for water the critical temperature is and the associated critical pressure is 220 atmospheres. There is a distinct nomenclature used for systems with more than one phase. In this, the terms ``vapor'' and ``gas'' seem to be used interchangeably. In the zone where both liquid and vapor exist, there are two bounding situations. When the last trace of vapor condenses, the state becomes saturated liquid. When the last trace of liquid evaporates the state becomes saturated vapor (or dry vapor). If we put heat into a saturated vapor it is referred to as superheated vapor. Nitrogen at room temperature and pressure (at one atmosphere the vaporization temperature of nitrogen is 77 K) is a superheated vapor.

Figure 8.4: Constant pressure curves in

coordinates showing vapor dome

Figure 8.4 shows lines of constant pressure in temperature-volume coordinates. Inside the vapor dome the constant pressure lines are also lines of constant temperature. It is useful to describe the situations encountered as we decrease the pressure or equivalently increase the specific volume, starting from a high pressure, low specific volume state (the upper left-hand side of the isotherm in Figure 8.3). The behavior in this region is liquid-like with very little compressibility. As the pressure is decreased, the volume changes little until the boundary of the vapor dome is reached. Once this occurs, however, the pressure is fixed because the temperature is constant. As the piston is withdrawn, the specific volume increases through more liquid evaporating and more vapor being produced. During this process, since the expansion is isothermal (we specified that it was), heat is transferred to the system. The specific volume will increase at constant pressure until the right hand boundary of the vapor dome is reached. At this point, all the liquid will have been transformed into vapor and the system again behaves as a single-phase fluid. For water at temperatures near room temperature, the behavior would be essentially that of a perfect gas in this region. To the right of the vapor dome, as mentioned above, the behavior is qualitatively like that of a perfect gas.

Figure 8.5: Specific volumes at constant temperature and states within the vapor

dome in a liquid-vapor system Referring to Figure 8.5, we define notation to be used in what follows. The states and denote the conditions at which all the fluid is in the liquid state and the gaseous state respectively. The specific volumes corresponding to these states are

For conditions corresponding to specific volumes between these two values, i.e., for state , the system would exist with part of the mass in a liquid state and part of the mass in a gaseous (vapor) state. The average specific volume for this condition is We can relate the average specific volume to the specific volumes for liquid and vapor and the mass that exists in the two phases as follows. The total mass of the system is given by The volume of the system is

The average specific volume, of the system

, is the ratio of the total volume to the total mass

The fraction of the total mass in the vapor phase is called quality, and denoted by :

In terms of the quality and specific volumes, the average specific volume can be expressed as

In reference to Figure 8.6,

Figure 8.6: Liquid vapor equilibrium in a two-phase medium

8.2 Work and Heat Transfer with TwoPhase Media


We examine the work and heat transfer in quasi-static processes with two-phase systems. For definiteness, consider the system to be a liquid-vapor mixture in a container whose volume can be varied through movement of a piston, as shown in Figure 8.6. The system is kept at constant temperature through contact with a heat reservoir at temperature . The pressure is thus also constant, but the volume, , can change. For a fixed mass, the volume is proportional to the specific volume so that point in Figure 8.6 must move to the left or the right as changes. This implies that the amount of mass in each of the two phases, and hence the quality, also changes because mass is transferred from one phase to the other. We wish to find the heat and work transfer associated with the change in mass in each phase. The change in volume can be related to the changes in mass in the two phases as,

The system mass is constant changes We can define the quantity

so that for any

In terms of

the volume change of the system is

The work done is given by

The change in internal energy, , can be found as follows. The internal energy of the system can be expressed in terms of the mass in each phase and the specific internal energy (internal energy per unit mass, ) of the phase as,

Note that the specific internal energy of the two-phase system can be expressed in a similar way as the specific volume in terms of the quality and the specific internal energy of each phase: Writing the first law for this process:

The heat needed for the transfer of mass is proportional to the difference in specific enthalpy between vapor and liquid. The pressure and temperature are constant, so that the specific internal energy and the specific enthalpy for the liquid phase and the gas phase are also constant. For a finite change in mass from liquid to vapor, , therefore, the quantity of heat needed is

The heat needed per unit mass,

, for transformation between the two phases is

The notation

refers to the specific enthalpy change between the liquid state

and the vapor state. The expression for the amount of heat needed, , is a particular case of the general result that in any reversible process at constant pressure, the heat flowing into, or out of, the system is equal to the enthalpy change. Heat is absorbed if the change is from solid to liquid (heat of fusion), liquid to vapor (heat of vaporization), or solid to vapor (heat of sublimation). A numerical example is furnished by the vaporization of water at :

1. How much heat is needed per unit mass of fluid vaporized? 2. How much work is done per unit mass of fluid vaporized? 3. What is the change in internal energy per unit mass of fluid vaporized? In addressing these questions, we make use of the fact that problems involving heat and work exchanges in two-phase media are important enough that the values of the specific thermodynamic properties that characterize these transformations have been computed for many different working fluids. The values are given in SB&VW in Tables B.1.1 and B.1.2 for water at saturated conditions and in Tables B.1.3, B.1.4, and B.1.5 for other conditions, as well as for other working fluids, as well as in the Appendix. From these, for water:

At

, the vapor pressure is , is .

. and the specific

The specific enthalpy of the vapor, enthalpy of the liquid, , is

The difference in enthalpy between liquid and vapor, enough so that it is tabulated also. This is .

, occurs often

The specific volume of the vapor is of the liquid is .

and the specific volume

The heat input to the system is the change in enthalpy between liquid and vapor, , and is equal to .

The work done is

which has a value of

The change in internal energy per unit mass

can be found from

or from the tabulated values as . This is much larger than the work done. Most of the heat input is used to change the internal energy rather than appearing as work.

Muddy Points For the vapor dome, is there vapor and liquid inside the dome and outside is it just liquid or just gas? Is it interchangeable? Is it true for the plasma phase? (MP 8.1) What is ? How do we find it? (MP 8.2) lines in the diagram. (MP 8.3)

Reasoning behind the slopes for

For a constant pressure heat addition, why is What is latent heat? (MP 8.5) Why is a function of ? (MP 8.6)

? (MP 8.4)

8.3 The Carnot Cycle as a Two-Phase Power Cycle

[cycle in

coordinates]

[cycle in

coordinates]

[cycle in - coordinates] Figure 8.7: Carnot cycle with two-phase medium A Carnot cycle that uses a two-phase fluid as the working medium is shown below in Figure 8.7. Figure 8.7(a) gives the cycle in - coordinates, Figure 8.7(b) in - coordinates, and Figure 8.7(c) in - coordinates. The boundary of the region in which there is liquid and vapor both present (the vapor dome) is also indicated. Note that the form of the cycle is different in the and - representation; it is only for a perfect gas with constant specific heats that cycles in the two coordinate representations have the same shapes. The processes in the cycle are as follows: 1. Start at state with saturated liquid (all of mass in liquid condition). Carry out a reversible isothermal expansion to ( ) until all the liquid is vaporized. During this process a quantity of heat received from the heat source at temperature . 2. Reversible adiabatic (i.e., isentropic) expansion ( temperature to . Generally state both liquid and vapor. 3. Isothermal compression ( ) at per unit mass is ) lowers the

will be in the region where there is to state . During this

compression, heat per unit mass is rejected to the source at . 4. Reversible adiabatic (i.e., isentropic) compression ( ) in which the vapor condenses to liquid and the state returns to .

In the

diagram the heat received,

, is

and the heat rejected,

, is

. The net work is represented by

. The thermal efficiency is given by

In the - diagram, the isentropic processes are vertical lines as in the diagram. The isotherms in the former, however, are not horizontal as they are in the latter. To see their shape we note that for these two-phase processes the isotherms are also lines of constant pressure (isobars), since combined first and second law is . The

For a constant pressure reversible process, constant pressure line in - coordinates is thus,

. The slope of a

The heat received and rejected per unit mass is given in terms of the enthalpy at the different states as

The thermal efficiency is

or, in terms of the work done during the isentropic compression and expansion processes, which correspond to the shaft work done on the fluid and received by the fluid,

8.3.1 Example: Carnot steam cycle

Heat source temperature =

Heat sink temperature =

What is the (i) thermal efficiency and (ii) ratio of turbine work to compression (pump) work if 1. all processes are reversible? 2. the turbine and the pump have adiabatic efficiencies of 0.8? Neglect the changes in kinetic energy at inlet and outlet of the turbine and pump. 1. For the reversible cycle,

2. 3. To find the work in the pump (compression process) or in the turbine, we need to find the enthalpy changes between states and , , and the

change between and , . To obtain these the approach is to use the fact that during the expansion to find the quality at state and then, knowing the quality, calculate the enthalpy as . We know the conditions at state , where the fluid is all vapor, i.e., we know , , :

4. 5. We now need to find the quality at state , . Using the definition of , we

quality given in Section 8.1, and noting that obtain,

6.
o

The quantity temperature

is the mass-weighted entropy at state .

, which is at

o o

The quantity The quantity .

is the entropy of the liquid at temperature

is the entropy of the gas (vapor) at temperature

The quantity

at

We know:

The quality at state

is thus,

The enthalpy at state

is,

Substituting the values,

The turbine work/unit mass is the difference between the enthalpy at state and state ,

We can apply a similar process to find the conditions at state

We have given that state is

. Also

at

. The quality at

The enthalpy at state

is

The work of compression (pump work) is the numerical values,

. Substituting

The ratio of turbine work to compression work (pump work) is

We can check the efficiency by computing the ratio of net work to the heat input . Doing this gives, not surprisingly, the same value as the Carnot equation. 7. For a cycle with adiabatic efficiencies of pump and turbine both equal to 0.8 (non-ideal components), the efficiency and work ratio can be found as follows. We can find the turbine work using the definition of turbine and compressor adiabatic efficiencies. The relation between the enthalpy changes is

Substituting the numerical values, the turbine work per unit mass is . For the compression process, we use the definition of compressor (or pump) adiabatic efficiency:

The value of the enthalpy at state efficiency is given by

is

. The thermal

Substituting the numerical values, we obtain for the thermal efficiency with non-ideal components, A question arises as to whether the Carnot cycle can be practically applied for power generation. The heat absorbed and the heat rejected both take place at constant temperature and pressure within the two-phase region. These can be closely approximated by a boiler for the heat addition process and a condenser for the heat rejection. Further, an efficient turbine can produce a reasonable approach to reversible adiabatic expansion, because the steam is expanded with only small losses. The difficulty occurs in the compression part of the cycle. If compression is carried out slowly, there is equilibrium between the liquid and the vapor, but the rate of power generation may be lower than desired and there can be appreciable heat transfer to the surroundings. Rapid compression will result in the two phases coming to very different temperatures (the liquid temperature rises very little during the compression whereas the vapor phase temperature changes considerably). Equilibrium between the two phases cannot be maintained and the approximation of reversibility is not reasonable. Another circumstance is that in a Carnot cycle all the heat is added at the same temperature. For high efficiency we need to do this at a higher temperature than the critical point, so that the heat addition no longer takes place in the two-phase region. Isothermal heat addition under this circumstance is difficult to accomplish. Also, if the heat source and the cycle are considered together, the products of combustion which provide the heat can be cooled only to the highest temperature of the cycle. The source will thus be at varying temperature while the system requires constant temperature heat addition, so there will be irreversible heat transfer. In summary, the practical application of the Carnot cycle is limited because of the inefficient compression process, the low work per cycle, the upper limit on temperature for operation in the two-phase flow regime, and the irreversibility in the heat transfer from the heat source. In the next section, we examine the Rankine cycle, which is much more compatible with the characteristics of two-phase media and available machinery for carrying out the processes.

Muddy Points What is the reason for studying two-phase cycles? (MP 8.7) How did you get thermal efficiency? How does a boiler work? (MP 8.8)

8.4 The Clausius-Clapeyron Equation (application of 1st and 2nd laws of thermodynamics)
Until now we have only considered ideal gases and we would like to show that the properties , , , etc. are true state variables and that the 1st and 2nd laws of thermodynamics hold when the working medium is not an ideal gas (i.e. a twophase medium). An elegant way to do this is to consider a Carnot cycle for a twophase medium. To state the fact that all Carnot engines operated between two given temperatures have the same efficiency is one way of stating the 2 nd law of thermodynamics. The working fluid need not be an ideal gas and may be a twophase medium changing phases. The idea is to run a Carnot engine between temperatures and for a two-phase medium and to let it undergo a change in phase. We can then derive an important relation known as the Clausius-Clapeyron equation, which gives the slope of the vapor pressure curve. We could then measure the vapor pressure curve for various substances and compare the measured slope to the Clausius-Clapeyron equation. This can then be viewed as an experimental proof of the general validity of the 1st and 2nd laws of thermodynamics!

Figure 8.8: Carnot cycle devised to test the validity of the laws of thermodynamics Consider the infinitesimal Carnot cycle shown in Figure 8.8. Heat is absorbed between states and . To vaporize an arbitrary amount of mass, the amount of heat

(8..1)

must be supplied to the system. From the 1st and 2nd laws of thermodynamics the thermal efficiency for a Carnot cycle can be written as

Hence, for the infinitesimal cycle considered above, (8..2)

The work along and between the work along by the rectangle :

nearly cancel such that the net work is the difference and , and can be viewed as the area enclosed

(8..3)

Substituting Equations (8.1) and (8.3) into (8.2) one obtains

Rearranging terms yields the Clausius-Clapeyron equation, which defines the slope of the vapor pressure curve: (8..4)

The beauty is that we have found a general relation between experimentally measurable quantities from first principles (1 st and 2nd laws of thermodynamics). In order to plot the Clausius-Clapeyron relation and to compare it against experimentally measured vapor pressure curves, we need to integrate Equation (8.4). To do so, the heat of vaporization and the specific volumes must be known functions of temperature. This is an important problem in physical chemistry but we shall not pursue it further here except to mention that if

variations in heat of vaporization can be neglected, the vapor phase is assumed to be an ideal gas, and

the specific volume of the liquid is small compared to that of the vapor phase,

the integration can be readily carried out8.1. Making these approximations, the Clausius-Clapeyron equation becomes

Carrying out the integration, the resulting expression is

Note that the vapor pressure curves are straight lines if

is plotted versus

and that the slope of the curves is , directly related to the heat of vaporization. Figures 8.9, 8.9, and 8.22 depict the vapor pressure curves for various substances. The fact that all known substances in the two-phase region fulfill the Clausius-Clapeyron equation provides the general validity of the 1st and 2nd laws of thermodynamics!

Figure 8.9: Clausius-Clapeyron Experimental Proof (1) [Gyarmathy, Thermodynamics I, Eidgen ssische Technische Hochschule Z rich, 1992]

Figure 8.10: Clausius-Clapeyron Experimental Proof (2) [Mahan, Elementary Chemical Thermodynamics, 1963]

8.5 Rankine Power Cycles

Figure 8.11: Rankine power cycle with two-phase working fluid [Moran and Shapiro, Fundamentals of Engineering Thermodynamics] A schematic of the components of a Rankine cycle is shown in Figure 8.11. The cycle is shown on - , - , and - coordinates in Figure 8.12. The processes in the Rankine cycle are as follows: : Cold liquid at initial temperature is pressurized reversibly to a high pressure by a pump. In this process, the volume changes slightly. 2. : Reversible constant pressure heating in a boiler to temperature 1. .

: Heat added at constant temperature (constant pressure), with transition of liquid to vapor. 4. : Isentropic expansion through a turbine. The quality decreases 3. from unity at point to 5. heat. . by extracting

: Liquid-vapor mixture condensed at temperature

coordinates]

coordinates]

[ - coordinates] Figure 8.12: Rankine cycle diagram. Stations correspond to those in Figure 8.11 In the Rankine cycle, the mean temperature at which heat is supplied is less than the maximum temperature, , so that the efficiency is less than that of a Carnot cycle working between the same maximum and minimum temperatures. The heat absorption takes place at constant pressure over , but only the part is isothermal. The heat rejected occurs over ; this is at both constant temperature and pressure. To examine the efficiency of the Rankine cycle, we define a mean effective temperature, , in terms of the heat exchanged and the entropy differences:

The thermal efficiency of the cycle is

The compression and expansion processes are isentropic, so the entropy differences are related by The thermal efficiency can be written in terms of the mean effective temperatures as

For the Rankine cycle, , . From this equation we see not only the reason that the cycle efficiency is less than that of a Carnot cycle, but the direction to move in terms of cycle design (increased increase the efficiency. ) if we wish to

There are several features that should be noted about Figure 8.12 and the Rankine cycle in general: 1. The - and the - diagrams are not similar in shape, as they were with the perfect gas with constant specific heats. The slope of a constant pressure reversible heat addition line is, as derived in Chapter 6,

In the two-phase region, constant pressure means also constant temperature, so the slope of the constant pressure heat addition line is constant and the line is straight. 2. The effect of irreversibilities is represented by the dashed line from to .

Irreversible behavior during the expansion results in a value of entropy at the end state of the expansion that is higher than . The enthalpy at the end of the expansion (the turbine exit) is thus higher for the irreversible process than for the reversible process, and, as seen for the Brayton cycle, the turbine work is thus lower in the irreversible case. 3. The Rankine cycle is less efficient than the Carnot cycle for given maximum and minimum temperatures, but, as said earlier, it is more effective as a practical power production device.

Muddy Points Where does degrees Rankine come from? Related to Rankine cycles? (MP 8.9)

8.6 Enhancements of, and Effect of Design Parameters on, Rankine Cycles
The basic Rankine cycle can be enhanced through processes such as superheating and reheat. Diagrams for a Rankine cycle with superheating are given in Figure 8.13. The heat addition is continued past the point of vapor saturation, in other words the vapor is heated so that its temperature is higher than the saturation temperature associated with . This does several things. First, it increases the mean temperature at which heat is added, , thus increasing the efficiency of the cycle. Second is that the quality of the two-phase mixture during the expansion is higher with superheating, so that there is less moisture content in the mixture as it flows through the turbine. (The moisture content at is less than that at .) This is an advantage in terms of decreasing the mechanical deterioration of the blading.

coordinates]

coordinates]

[ - coordinates] Figure 8.13: Rankine cycle with superheating The heat exchanges in the superheated cycle are:

Along

, which is a constant pressure (isobaric) process: .

Along

The thermal efficiency of the ideal Rankine cycle with superheating is

This can be expressed explicitly in terms of turbine work and compression (pump) work as

Compared to the basic cycle, superheating has increased the turbine work, increased the mean temperature at which heat is received, the cycle efficiency. , and increased

Figure 8.14: Comparison of Rankine cycle with superheating and Carnot cycle

Figure 8.15: Rankine cycle with superheating and reheat for space power application A comparison of the Carnot cycle and the Rankine cycle with superheating is given in Figure 8.14. The maximum and minimum temperatures are the same, but the average temperature at which heat is absorbed is lower for the Rankine cycle. To alleviate the problem of having moisture in the turbine, one can heat again after an initial expansion in a turbine, as shown in Figure 8.15, which gives a schematic of a Rankine cycle for space power application. This process is known as reheat. The main practical advantage of reheat (and of superheating) is the decrease in moisture content in the turbine because most of the heat addition in the cycle occurs in the vaporization part of the heat addition process.

Figure 8.16: Effect of exit pressure on Rankine cycle efficiency We can also examine the effect of variations in design parameters on the Rankine cycle. Consider first the changes in cycle output due to a decrease in exit pressure. In terms of the cycle shown in Figure 8.16, the exit pressure would be decreased from to . The original cycle is , and the

modified cycle is . The consequences are that the cycle work, which is the integral of around the cycle, is increased. In addition, as drawn, although the levels of the mean temperature at which the heat is

absorbed and rejected both decrease, the largest change is the mean temperature of the heat rejection, so that the thermal efficiency increases.

Figure 8.17: Effect of maximum boiler pressure on Rankine cycle efficiency Another design parameter is the maximum cycle pressure. Figure 8.17 shows a comparison of two cycles with different maximum pressure but the same maximum temperature, which is set by material properties. The average temperature at which the heat is supplied for the cycle with a higher maximum pressure is increased over the original cycle, so that the efficiency increases.

Muddy Points Why do we look at the ratio of pump (compression) work to turbine work? We did not do that for the Brayton cycle. (MP 8.10) Shouldn't the efficiency of the super/re-heated Rankine cycle be larger because its area is greater? (MP 8.11) Why can't we harness the energy in the warm water after condensing the steam in a power plant? (MP 8.12)

8.7 Combined Cycles in Stationary Gas Turbine for Power Production


The turbine entry temperature in a gas turbine (Brayton) cycle is considerably higher than the peak steam temperature. Depending on the compression ratio of the gas turbine, the turbine exhaust temperature may be high enough to permit efficient generation of steam using the ``waste heat'' from the gas turbine. A configuration such as this is known as a gas turbine-steam combined cycle power plant. The cycle is illustrated in Figure 8.18.

Figure 8.18: Gas turbine-steam combined cycle [Kerrebrock, Aircraft Engines and Gas Turbines]

Figure 8.19: Schematic of combined cycle using gas turbine (Brayton cycle) and steam turbine (Rankine cycle) [Langston] The heat input to the combined cycle is the same as that for the gas turbine, but the work output is larger (by the work of the Rankine cycle steam turbine). A schematic of the overall heat engine, which can be thought of as composed of an upper and a lower heat engine in series, is given in Figure 8.19. The upper engine is the gas turbine (Brayton cycle) which expels heat to the lower engine, the steam turbine (Rankine cycle). The overall efficiency of the combined cycle can be derived as follows. We denote the heat received by the gas turbine as and the heat rejected to the

atmosphere as . The heat out of the gas turbine is denoted as . The hot exhaust gases from the gas turbine pass through a heat exchanger where they are used as the heat source for the two-phase Rankine cycle, so that is also the heat input to the steam cycle. The overall combined cycle efficiency is

where the subscripts refer to combined cycle (CC), Brayton cycle (B) and Rankine cycle (R) respectively. From the first law, the overall efficiency can be expressed in terms of the heat inputs and heat rejections of the two cycles as (using the quantity the magnitude of the heat transferred): to denote

The first square bracket term on the right hand side is the Brayton cycle efficiency, , the second is the Rankine cycle efficiency, , and the term in

parentheses is

. The combined cycle efficiency can thus be written as (8..5)

Equation (8.5) gives insight into why combined cycles are so successful. Suppose that the gas turbine cycle has an efficiency of 40%, which is a representative value for current Brayton cycle gas turbines, and the Rankine cycle has an efficiency of 30%. The combined cycle efficiency would be 58%, which is a very large increase over either of the two simple cycles. Some representative efficiencies and power outputs for different cycles are shown in Figure 8.20.

Figure 8.20: Comparison of efficiency and power output of various power products [Bartol (1997)]

8.8 Some Overall Comments on Thermodynamic Cycles


1. There are many different power and propulsion cycles, and we have only looked at a few of these. Many other cycles have been devised in the search for ways to increase efficiency and power in practical devices. 2. We can view a given cycle in terms of elementary Carnot cycles, as sketched in Figure 6.5. This shows that the efficiency of any other cycle operating between two given temperatures will be less than that of a Carnot cycle. 3. If we view the thermal efficiency as

(derived in Section 8.5), this means that we should accept heat at a high temperature and reject it at a low temperature for high efficiency. This objective must be tempered by considerations of practical application. 4. The cycle diagrams in - and - coordinates will only be similar if the working medium is an ideal gas. For other media (for example, a twophase mixture) they will look different. 5. Combined cycles make use of the rejected heat from a ``topping'' cycle as heat source for a ``bottoming'' cycle. The overall efficiency is higher than the efficiency of either cycle.

8.9 Muddiest Points on Chapter 8


MP 8..1 For the vapor dome, is there vapor and liquid inside the dome and outside is it just liquid or just gas? Is it interchangeable? Is it true for the plasma phase? The vapor dome separates the two-phase region from the single-phase region. Inside, we have a mixture of liquid and vapor. The peak of the vapor dome is called the critical point. The left-hand side leg of the vapor dome (from the critical point) is called the saturated liquid line along which the quality is zero (purely liquid). The right-hand side leg is denoted the saturated vapor line and the quality is one (purely vapor). For further details see the notes. Heating of a solid or liquid substance leads to phase transition to a liquid or gaseous state, respectively. This takes place at a constant temperature for a given pressure, and requires an amount of energy known as latent heat. On the

other hand, the transition from a gas to an ionized gas, i.e., plasma, is not a phase transition, since it occurs gradually with increasing temperature. During the process, a molecular gas dissociates first into an atomic gas which, with increasing temperature, is ionized. The resulting plasma consists of a mixture of neutral particles, positive ions (atoms or molecules that have lost one or more electrons), and negative electrons. MP 8..2 What is ? How do we find it?

The quantity represents the specific enthalpy change between the liquid and vapor phases of a substance at constant temperature, and thus constant pressure. It is therefore the heat input, per unit mass, to vaporize a kilogram of liquid. See notes, Section 8.2. MP 8..3 Reasoning behind the slopes for diagram. lines in the -

The slope of an isotherm in the gaseous phase (to the right of the vapor dome) is similar to the slope we found for the isotherm of an ideal gas . Inside the vapor dome pressure and temperature are directly related to one another ( , vapor pressure curve) such that an isotherm is a horizontal line (isobar inside the vapor dome). In the liquid phase the isotherms are very steep lines, because for liquids the volume is about constant (very low compressibility). MP 8..4 For a constant pressure heat addition, why is The combined first and second law is constant pressure process, ?

. For a reversible , is . Thus for a

, and the heat input, .

reversible constant pressure process,

For an irreversible process we can say from the steady flow energy equation: . For a steady flow, the one-dimensional momentum equation is

where represents the viscous forces in an irreversible flow. Combining these two expressions, and using (the condition of constant pressure) gives

Without going into any detail concerning the form of the viscous forces, this equation shows that the equality between heat input and enthalpy change does not hold for general irreversible flow processes at constant pressure. MP 8..5 What is latent heat? Latent heat is a term for the enthalpy change needed for vaporization. MP 8..6 Why is a function of ?

Inside the vapor dome we have a mixture of liquid and vapor. The internal energy of the system (liquid and vapor) can be expressed in terms of the mass in each phase and the specific internal energy of each phase as,

Introducing the quality

as the fraction of the total mass in the vapor phase, , we can write

Since the specific energy of the saturated liquid and the saturated vapor are functions of temperature the internal energy of the two-phase system is a function of and (see also notes Section 8.2). MP 8..7 What is the reason for studying two-phase cycles? We study them because of their immense practical utility in a number of industrial devices and their intrinsic interest as applications of the basic principles. MP 8..8 How did you get thermal efficiency? How does a boiler work? The thermal efficiency is, as previously, the net work done divided by the heat input. Using the first law for a control volume we can write both of these quantities in terms of the enthalpy at different states of the cycle.

For the steam cycles discussed in class, a boiler is a large (as in the viewgraph of the Mitsubishi power plant) structure with a lot of tubes running through it. The water (or whatever medium is used in the cycle) runs through the tubes. Hot gases wash over the outside of the tubes. The hot gases could be from a combustor or from the exit of a gas turbine. MP 8..9 Where does degrees Rankine come from? Related to Rankine cycles? I think the answer is yes, although I do not know for sure. If so, this is the same Rankine who has his name on the Rankine-Hugoniot conditions across a shock wave. MP 8..10 Why do we look at the ratio of pump (compression) work to turbine work? We did not do that for the Brayton cycle. If the ratio of compression work to turbine work were close to unity for an ideal cycle, small changes in component efficiencies would have large effects on cycle efficiency and work. For the Rankine cycle this is not true. (The effect of pump efficiency on Rankine cycle efficiency is clearly small in the class example.) For the Brayton cycle, where the net work is the difference of two numbers which are of (relatively) similar sizes, the effect of compressor and turbine efficiency on cycle efficiency can be much larger. I used the word ``sensitive'' and the meaning was that the cycle performance responded strongly to changes in the compressor and turbine behavior. MP 8..11 Shouldn't the efficiency of the super/re-heated Rankine cycle be larger because its area is greater? The area enclosed by an ideal cycle in a - diagram is the net work done, but it does not tell you about efficiency. We saw that for example when we looked at the Brayton cycle for the condition of maximum work, rather than maximum efficiency (see Section 3.7.2). MP 8..12 Why can't we harness the energy in the warm water after condensing the steam in a power plant? Let's assume the temperature of the warm water after condensing the steam is at a temperature of about 30 to 40 degrees C. If we consider running a heat engine between this heat reservoir (say 35 degrees C) and the surroundings at 20 degrees C, we would get an ideal thermal efficiency of about 5%. In other words, the available useful work is relatively small if we considered the lower heat reservoir to be the surroundings. In general, there is a property that only depends on state variables called availability. The change in availability gives

the maximum work between two states, where one state is referred to the surroundings (dead state).

Figure 8.21: An industrial

diagram

Figure 8.22: Clausius-Clapeyron Experimental Proof (3) [Burton Corblin North America]

You might also like